Clinical Immunology and Allergy

You might also like

Download as pdf or txt
Download as pdf or txt
You are on page 1of 156

Clinical Immunology and Allergy

A 19-year-old male patient presented with lymphocytic interstitial pneumonia. Lymph nodes
histopathological examination exhibited an effaced structure and a diffuse infiltration of small-sized
lymphocytic cells compatible with a low-grade non-Hodgkin's lymphoma. The surface phenotype
of the sample taken corresponded to sample 4.
Choose the correct cell type for sample 4.

1)
2)
3)
4)
5)

A mature, MHC class I restricted T-cell


A mature, MHC class II restricted T-cell
A mature B-cell
An immature T-cell
An immature B-cell

The correct answer is choice D.

Thymocytes fall into 4 classes, CD4-, CD8- (immature T-cells); CD4+, CD8+ (immature T-cells);
CD4-, CD8+ (mature cytotoxic T-cells) ; and CD4+, CD8- (mature helper T-cells), which are
sometimes called double negative, double positive, and single positive T cells, respectively. Double
negative cells can differentiate into the other kinds of cells, but that the single positive cells are
terminally differentiated. T-cell receptor (TCR) complex contains an accessory molecule, called
CD3, that is essential of signal transduction. All T cells are positive for CD3. Thus sample 4 is a
CD4+, CD8+(double positive) immature T cell. Hence the correct answer is choice D.

T cell receptor protein doesnt bind directly to the antigen. It only can bind to pieces of antigen
molecules on the surface of antigen-presenting cells (APCs). MHC proteins serve as carriers of
antigenic material on the surface of APCs. In humans, the MHC gene loci is divided into three
regions: Class I, II, and III. Class I and II genes mostly encode cell surface proteins. Class III genes
encode secreted and intracellular proteins. Cytotoxic T CD4-, CD8+ cells mostly recognize bits of
antigen in association with Class I MHC proteins (choice A), while helper T CD4+, CD8- cells
mostly bind to antigen fragments associated with Class II MHC proteins (choice B).

Immature B cells express IgM (choice E) while mature B cells express both IgM and IgD on their
surface (choice C).

T-cell development in thymus

During initial development of T and B cells in the primary lymphoid organs--such as thymus and
bone marrow-- many of the self-reactive and useless clones are actually killed off so that they no
longer exist. This process is called selection. Most of the thymocytes (immature T cells) that enter
the thymus die there and only about 2% survive and leave the thymus as mature T cells. That
massive cell death occurs due to selection of the right T cells. Positive selection results in MHC
restriction and negative selection results in self tolerance.

1. Pre-T cells leave the bone marrow and migrate to the thymus.
2. Upon reaching the thymus they begin to undergo rearrangement of their and then TCR
genes and to express surface TCR. Cells that fail to do this die.
3. Some T cells will express receptors that cannot bind at all to self-MHC proteins. These cells will
die; i.e., only T cells with receptors that bind self-MHC will survive (positive selection to generate
competent T cells ).
4. Some surviving T cells will express receptors that bind very strongly to self-MHC proteins
on the surface of APC in the thymus. These T cells will also die (negative selection to eliminate

autoimmunity).
5. Cells that finally survive become mature T cells and exit the thymus into the peripheral
circulation.

Theres a progession of developmental stages as T cells mature through the thymus, and they
change their surface molecules as they pass through these stages. The cells entering the thymus
express TCR proteins and CD3. Most first rearrange their beta TCR chain and generate a cell
surface pseudoTCR containing a beta chain and pre-alpha chain plus CD3. These pre or
immature T cells are CD4-, CD8-. Subsequently, they switch on both CD4+, CD8+, and the alpha
chain of the TCR undergoes rearrangement. These immature double positive cells undergo positive
and negative selection, and those that survive undergo a final selective step whereby they lose
either CD4 or CD8 and become single-positive T cells that will be either functional T-helper or Tcytotoxic cells respectively. TCR rearrangement and positive selection occurs in the thymic cortex
while negative selection occurs in the thymic medulla .

A 40 year old zoo employee was admitted to the emergency after being bitten by a Gaboon Viper
on his hand. He complained of nausea and had an episode of vomiting as well. He was unable to
move his hands and edema was observed over the bitten area. Prothrombin time was 15.53
(normal range 11.0-13.0). International normal ratio (INR) was measured to be 1.2. He was
administered intravenous antibiotics and antivenin was also given. Local debridement and

fasciotomy were performed the next day.


Administration of antivenin is an example of which of the following:
1)
2)
3)
4)
5)

IgA mediated mucosal immune response to the antigen


Acquired passive immunity
Vaccination against snake venom
Anti allergic measures
IgE mediated immunity

The correct answer is choice B

Acquired passive immunity refers to the process of obtaining serum from immune individuals,
concentrating the immunoglobulin fraction to protect a susceptible person. Passive immunity gives
immediate, but short-lived protection. The method of acquired passive immunization is used to
combat active infections of tetanus, infections hepatitis, rabies, snake bit venom etc. In the case of
snake bite venom, passive immunity is produced by the antitoxin so the serum is called antivenom
serum/antivenin.

The Mucosal immune system (choice A) consists of an integrated network of lymphoid tissues and
mucous membrane-associated cells and effector molecules. It provides the host protection from
environmental antigens and pathogens. The Mucosal immune system includes antibodies, largely

of IgA isotype, as well as cytokines, chemokines, and their receptors and immunocytes that
accumulate in, or in transit between, various mucosa-associated lymphoid tissues (MALT).

Vaccination (choice C) builds up immunity against a particular antigen/pathogen. Administration of


antivenin only provides immediate clearance of the venom by providing polyvalent antisera (usually
equine) that was exposed to small doses of snake venom.

Snake venom contains a mixture of proteins and enzymes such as hyaluronidase,


phosphodiesterase, ATPases and phospholipase. It is actively cytotoxic and the symptoms are due
to these toxins and not a allergic reaction (choice D).

IgE antibodies (choice E) bind to allergens to cause release of histamines.

Patients suffering from end stage renal failure who are on chronic intermittent hemodialysis (HD),
show an immunodeficiency of both cell mediated and humoral type. It is manifested in the form of
frequent infections, higher incidences of mortality due to infectious complications and low response
to vaccination.
This immune defect is the result of defective T- cell activation and is NOT the result of which of the

following?
1)
2)
3)
4)
5)

Absence of co-stimulatory signals


Aberrant secretion of cytokines IL-2 and TNF-
Injury to lymphoid tissue due to uremia
Defect in Antigen presenting cells (APCs)
Complement activation

The correct answer is choice C.

In patients undergoing HD, both reduced co-stimulation and aberrant expression of cytokines has
been observed. This is mainly induced by uremia and further exacerbated by regular contact of
blood with artificial dialyzer membranes, complement activation and recurrent exposure to
pyrogens. The HD patients are thought to be in a chronic state of inflammation. The cytokine
differentiation profile of circulating T cells from HD patients has been found to be dysregulated and
is characterized by an increase in TH1 cells, but a normal amount of TH2 cells. Moreover, a higher
percentage of monocytes capable of secreting the TH1 promoting cytokine interleukin 12 (IL-12)
compared to normal populations are observed. There is no documented research correlating
lymphoid tissue damage to uremia.

A defect in the Antigen presenting cells (APCs) results in their exhibiting a decreased co-stimulatory
activity. This defect can be abrogated in vitro by the application of a co-stimulatory signal of the
B7/CD28 pathway (choice A).

In patients on hemodialysis, pro-inflammatory cytokines such as IL-6, TNF- and IL-1 are
increased due to the state of chronic inflammation (choice B). Thus, the defect in the T-cell
activation mainly rests in the responses of the Antigen presenting cells (choice D).

As mentioned earlier, regular contact of blood with artificial dialyzer membranes causes excessive
complement activation (choice E) which results in a state of chronic inflammation, ultimately
resulting in immune dysfunction.

Viruses employ various mechanisms to evade the host immune response. The Herpes simplex
virus encodes a protein called ICP47 (an immediate-early protein) that binds to the cytosolic face of
transporter for antigen processing (TAP) and blocks accessibility of TAP to peptides.
Thus, herpes virus infected cells that express ICP47 would undergo which of the following actions:
1)
2)
3)
4)
5)

Resist killing by CD8+ virus-specific cytotoxic T lymphocytes


Be unable to stimulate virus-specific antibody production by B lymphocytes
Not stimulate CD4+ response
Resist complement mediated lysis of the immune complex
Avoid detection by macrophages

The correct answer is choice A.

The herpes simplex viruses (HSV) avoid host responses by inhibition of antigen presentation by
infected host cells. HSV-1 and HSV-2 both express a protein that is synthesized shortly after viral
replication called ICP47, which very effectively inhibits the human transporter molecule needed for
antigen processing. Inhibition of TAP blocks antigen delivery to class I MHC receptors on HSVinfected cells. This prevents the presentation of viral antigen to CD8+ T cells resulting in the
trapping of empty class I MHC molecules in the endoplasmic reticulum and effectively shutting
down the CD8+ T-cell response to HSV-infected cells.

When the above mentioned response is inhibited, the antibodies would not recognize the herpes
antigen and further antibody production (Choice B).

The herpes virus evades the CTL response, via class I MHC, and not through the CD4+ /class II
MHC response (choice C).

Antibody-mediated destruction of viruses requires complement activation, that results either in


direct lysis of the viral particle or opsonization and elimination of the virus by phagocytic cells. In

this case, the complement system cannot detect any immune complex, which is not formed (choice
D).

Since herpes virus is an intracellular pathogen, the mechanism of immune response against it does
not involve the macrophages (choice E) that detect extracellular pathogens and engulf them.
A 3 month old baby girl was admitted for work-up for loose stools and rectal bleeding. Small
patches of eczema were seen on her body. Some form of food allergy was suspected by the
admitting physician. The baby had been weaned from breast milk a few weeks ago and was on
formula milk and semisolid food containing dairy products. She was treated conservatively and
improved soon after her admission. The parents had a family history of allergies and asthma. They
were suitably advised on prevention of food allergies in babies and discharged.
Which of the following is a current recommendation of the American Academy of Pediatrics (AAP)
for decreasing the incidence of food allergy in at-risk children?
1)
2)
3)
4)
5)

Breast feed exclusively for 12 months


If unable to breast feed, use soy-based formulas rather than cows milk formula
Include peanut in the lactation diet to induce oral tolerance
Delay introduction of solid foods
Begin feeding whole cows milk at age 4 months, before allergies develop

The correct answer is choice D.

The American Academy of Pediatrics (AAP) has put forth some guidelines that, over large
population groups, are believed to help prevent food allergies in high-risk children (those with
parents or siblings with food allergies or other allergic disorders, like eczema). According to the
guidelines, it is advisable to wait until infants are four to six months old to introduce solid foods.

Breastfeeding exclusively for at least four to six months (not 12 months), if possible, also reduces
the risk of food allergies (choice A).

If breastfeeding isn't possible or workable in some circumstances, hydrolysate formulas - formulas


that have been broken down into simpler proteins that are considered easier to digest are
advisable. At this time, extensively hydrolyzed formulas are considered the most effective for
preventing allergic conditions. Soy formulas and goat's milk may not be good alternatives, because
many infants that are allergic to cow's milk may also be allergic to soy) (choice B).

Breastfeeding mothers, should also avoid cows milk, eggs, fish, peanuts, and tree nuts from their
own diet, consumption of peanuts by the lactating mother is not advised (choice C). Cows milk
should be avoided in infants till the age of one year (choice E).

A 29-year-old woman was diagnosed as having type 1 diabetes mellitus and received insulin aspart
and neutral protamine hagedorn (NPH) insulin. On day 22, she had leg edema and right abdominal
pain. The serum hepatobiliary enzyme levels were markedly elevated. Computed tomography
revealed gallbladder edema. There was no skin eruption at the local injection site. Oral intake of
foods and drinks was ceased and she was given total parenteral nutrition and human regular
insulin. Challenge testing demonstrated that protamine was the cause of her allergy. This is an
extremely rare case of protamine allergy resulting in gallbladder edema.
What type of hypersensitivity reaction has this patient experienced?
1)
2)
3)
4)
5)

Type III
Type Iva
Type Ivb
Type Ivc
Type Ivd

The correct answer is choice D.

The hallmarks of a type IV reaction are the delay in time required for the reaction to develop. A type
IV hypersensitive reaction involves the cell-mediated branch of the immune system.

Type IVc: T cells can also act as effector cells themselves: they emigrate to the tissue and can kill
tissue cells like hepatocytes or keratinocytes in a perforin/granzyme B- and FasL-dependent
manner. Such reactions occur in most drug-induced delayed hypersensitivity reactions, mostly
together with other type IV reactions (monocyte, eosinophil or PMN recruitment and activation).
Cytotoxic T cells thus play a role in maculopapular or bullous skin diseases as well as in
neutrophilic inflammations (acute generalized exanthematous pustulosis AGEP), and in contact
dermatitis. Type IVc reactions appear to be dominant in bullous skin reactions, where activated
CD8+ T cells kill keratinocytes, but may also be the dominant cell type in hepatitis or nephritis. In
this case, the gall bladder edema along with hepatic dysfunction point to a type IVc DTH.

Type IVa (choice B) reactions correspond to TH1-type immune reactions: TH1-type T cells activate
macrophages by secreting large amounts of interferon (IFN)- , drive the production of complement
fixing antibody isotypes involved in type II and III reactions (IgG1, IgG3), and are co-stimulatory for
pro-inflammatory responses (TNF, IL-12) and CD8+ T-cell responses. The T cells promote these
reactions by IFN- secretion and possibly other cytokines (TNF, IL-18, etc.).

Type IVb (choice C) corresponds to the TH2-type immune response. TH2 T cells secrete the
cytokines IL-4, IL-13 and IL-5, which promote B-cell production of IgE and IgG4, macrophage
deactivation and mast cell and eosinophil responses. The high production of the Th2 cytokine IL-5
leads to an eosinophilic inflammation, which is the characteristic inflammatory cell type in many
drug hypersensitivity reactions. In addition, there is a link to type I reactions, as Th2 cells boost IgE
production by IL-4/IL-13 secretion. Examples include infestations with nematodes, or an allergic
inflammation of the bronchi or nasal mucosa (asthma and rhinitis).

Type IVd: T cells can coordinate (sterile) neutrophilic inflammations as well. Typical examples
would be sterile neutrophilic inflammations of the skin, in particular AGEP. In this disease, CXCL8and GM-CSF-producing T cells recruit neutrophilic leukocytes via CXCL8 release and prevent their
apoptosis via GM-CSF release. Besides AGEP, such T-cell reactions are also found in Behets
disease and pustular psoriasis .

A type III hypersensitive reaction (choice A) is mediated by the formation of immune complexes and
the ensuing activation of complement. Complement split products serve as immune effector
molecules that elicit localized vasodilation and chemotactically attract neutrophils. Deposition of
immune complexes near the site of antigen entry can induce an Arthus reaction, in which lytic
enzymes released by the accumulated neutrophils and the complement membrane attack complex
and cause localized tissue damage.

Type I hypersensitivity reaction or allergy is immediate, in response to an antigen and is IgE


antibody mediated. Clinical manifestations of type I reactions include potentially life-threatening
systemic anaphylaxis and localized responses such as hay fever and asthma.

Type II or Cytotoxic Hypersensitivity also involves antibody-mediated reactions. However, the

immunoglobulin class (isotype) is generally IgG. Type II hypersensitivity may also involve
complement that binds to cell-bound antibody. The difference here is that the antibodies are
specific for (or able to cross-react with) "self" antigens. When these circulating antibodies react with
a host cell surface, tissue damage may result for e.g. autoimmune hemolytic anemia (AHA).

Two polio vaccines are used throughout the world to combat poliomyelitis (or polio). The two
vaccines have eradicated polio from most countries in the world, and reduced the worldwide
incidence from an estimated 350,000 cases in 1988 to 1,652 cases in 2007.
All of the following groups are contraindicated to receive the polio vaccine except?
1) A 45 year old man who has previously had streptomycin allergy
2) A 12 month old girl who is experiencing a kidney infection
3) A 12 month old boy who exhibited allergy symptoms after receiving his polio shot at 4
months
4) A 7 year old girl who developed allergy to neomycin when she was 6
5) A 32 year old woman who is HIV positive but clinically healthy

The correct answer is choice E.

Current US recommendations are to vaccinate children and specific groups of adults with the

inactivated polio vaccine (IPV). The oral polio vaccine has not been recommended in the US since
2000 due to rare cases of vaccinated individuals contracting polio from the vaccine. Children should
receive a series of 4 shots at the ages of:

2 months

4 months

6 18 months

4 6 years

Adults recommended to receive vaccination are those who:

Expect to travel to areas where polio is endemic

Are laboratory workers who may handle the virus

Are healthcare workers who may treat polio patients.

IPV is contraindicated in individuals who have previously shown allergy symptoms to streptomycin,
polymyxin B, neomycin, or a previous polio shot. Individuals who are moderately or severely ill
should wait until they recover to receive the vaccine.

HIV infection (choice E) is not a contraindication for polio vaccine.

A 48-year-old man presents to your clinic complaining of a chronic cough with fever that he has had
for the last 2 months. On physical examination, his temperature is 37.9 C. You order a chest x-ray
which reveals a diffuse bilateral reticulonodular pattern. You decide to refer him to a thoracic

surgeon and a transbronchial biopsy is performed. On microscopic examination of the biopsy, there
are focal areas of inflammation containing epithelioid macrophages, Langhans giant cells, and
lymphocytes. Findings are consistent with Granuloma formation with reactivation or reinfection
tuberculosis.
These findings are most typical for which of the following immunologic responses?
1) Exaggerated cellular immune response
2) Type II hypersensitivity
3) Delayed type hyersensitivity
4) CD4+ Th2
5) Type IV hypersensitivity

The correct answer is choice A.

Granulomas form as the result of a heightened cellular immune response when the body attempts
to isolate foreign substances that it cannot eliminate. The may be caused by infectious orgamisms,
most commonly mycobacteria or fungi, or non-infectious means leading to conditions such as
sarcoidosis. The inflammatory response leading to the formation of granulomas begins with
migration of mononuclear inflammatory cells to the site. Typically, CD4 Th1 and mononuclear
phagocytes are prominent early actors. Macrophages, epitheloid cells, and giant cells follow. The
area is frequently rimmed by T cells, primarily CD4 cells, and becomes interlaced with fibrin.

It is thought that the clinical effects of granuloma formation are due more to the physical effects of

the granuloma disrupting normal tissue architecture than to immunological processes following the
release of immune mediators. The following table reviews the incorrect choices in this question
(choices B, C and E).

Table 1. Comparison of Different Types of hypersensitivity reactions

A 28 year old male, who is a heavy smoker presented to your clinic with a chronic cough. He
complained of spitting up blood with coughing which developed about 1 month prior. The patient

was otherwise healthy, but concerned about contracting tuberculosis since he travelled frequently
to countries with high incidence of tuberculosis. A tuberculin skin test was performed that read at 72
hours as 5 mm.
This is a negative result in each of the following conditions except:
1)
2)
3)
4)
5)

Health care worker


Patient with Rheumatoid Arthritis
Patient on corticosteroids (10mg/day x 3 weeks)
A normal healthy individual
Previously normal tuberculin test (within 2 years)

The correct answer is choice B.

Tuberculin reaction is classified as positive based on the diameter of the induration in relation to
certain patient-specific risk factors. People with compromised immune functions will have a milder
reaction to tuberculin. A positive reaction to tuberculin may be the result of a previous natural
infection with M. tuberculosis, infection with a variety of non-tuberculosis mycobacteria (crossreaction), or tuberculosis vaccination with a live, but weakened (attenuated) mycobacterial strain.

Induration of less than 2 mm, without blistering, is considered a negative skin test. In patients who
are immunocompromised, such as people with HIV, rheumatoid arthritis, organ transplant cases

and Crohn's disease, 5 mm of induration is considered a positive skin test result. Also, patients who
may have contacted tuberculosis recently and persons with nodular or fibrotic changes on chest xray consistent with old healed TB, a 5 mm tuberculin test is considered positive. A person who
received a Bacille Calmette Guerin (BCG) vaccine that is administered in some countries, against
tuberculosis may also have a positive skin reaction to the TB test which is less than 10 mm
induration.

In people with underlying kidney disease, diabetes, or a health-care worker (choice A) or person
with personal contact of someone with active TB, 10 mm of induration is considered a positive skin
test. Moreover, 10 mm is positive in people who have arrived from high tuberculosis prevalence
countries (less than 5 years), Residents and employees of high-risk congregate settings (e.g.,
prisons, nursing homes, hospitals, homeless shelters, etc.) and patients with diabetes, prolonged
corticosteroid therapy (choice C), leukemia, end-stage renal disease, chronic malabsorption
syndromes etc.

In a healthy person whose immune system is normal, induration greater than or equal to 15 mm
(choice D) is considered a positive skin test. If blisters are present (vesiculation), the test is also
considered positive.

A tuberculin test conversion is defined as an increase of 10 mm or more within a 2-year period

(choice E), regardless of age.

In summary:

The antibodies primarily responsible for activating the classic complement cascade include which of
the following:
1) IgE
2) IgD
3) IgM, IgD
4) IgG, IgM
5) IgM

The correct answer is choice D

Antibodies are glycoproteins in nature. Papain digestion breaks the immunoglobulin molecule from
the hinge region, giving rise to two parts. One part is known as the Fab (fragment antigen binding).
The proteolytic fragments from the other part tends to crystallize and therefore it is known as the
fragment crystallization (Fc). Fc part is made up of constant regions of the two heavy chains. Based
on the differences of amino acids in the constant region of heavy chains, immunoglobulins can be
divided into five isotypes. The antigen binding region helps antibodies to recognize and bind to
various antigens while Fc portion can mount the necessary immune response in form of activating
various immune effector cells or by activating complement proteins.

Complement system is composed of a set of circulatory proteins and also the proteins attached to
the cell membranes. The function of the complement system is well described by its name as these
proteins assist or complement the antibodies in their antimicrobial activities. One pathway of
activating the complement system of proteins is through the microbes and is called as the

Alternative pathway. Second pathway is termed as the Classical pathway and it is activated through
the antibodies attached to the antigens. The third pathway leading to the complement activation is
termed as the lectin pathway.

The classical pathway is triggered when either IgM or certain subclasses of IgG (IgG1 and IgG3)
binds to the antigen or a microbe. When the two Fc region of the two antibodies come into a close
proximity to each other, then a C1protein is recruited and attaches to the Fc regions thus forming a
complex. C1 protein becomes enzymatically active and this results in a sequence of events leading
to the activation of other proteins of the complement system like C4, C2. The activation of
complement system can either cause direct lysis of the foreign cells or indirectly e.g. through
opsonization.

A 30 year old African-American was referred for immunological testing to ascertain the cause of her
meningitis following infection with Neisseria meningitidis. X-ray of the skull and sinuses was normal
and did not show any abnormal communication with the cerebrospinal fluid. Thus, a possible
immune defect was suspected.
Total and specific immunoglobulin levels were normal. Total classical pathway hemolytic
complement activity (CH50) and alternate pathway (AP 50) were undetectable. Tests revealed an
isolated deficiency of C6, with normal levels of all other components. Half normal levels of C6 were
found in the sera of her parents and in three of four siblings. Patients with C6 complement

deficiency are at higher risk of Neisseria infection.


The complement system provides immunity through which of the following functions?
1)
2)
3)
4)
5)

Class I recognition
IL-2 production
Activated T-cell response
Immune complex clearing
Activated B-cell response

The correct answer is choice D.

The complement proteins are a group of proteins that assist in eliminating foreign pathogens and in
promoting the ensuing inflammatory reaction. Complement system helps fight infection through
lysis of foreign cells by antibody-dependent or antibody-independent pathways; opsonization or
uptake of particulate antigens, including bacteria, by phagocytes; activation of inflammatory
responses; and clearance of circulating immune complexes by cells in the liver and spleen. The
complement protein- C-reactive protein binds to the C-polysaccharide cell-wall component found on
a variety of bacteria and fungi. This binding activates the complement system, resulting in
increased clearance of the pathogen either by complement-mediated lysis or by a complement
mediated increase in phagocytosis. Binding of antibody to antigen on a microorganism can also
activate the complement system, resulting in lysis of the foreign organism. Complement includes a

collection of proteins that can perforate cell membranes.

C6 deficiencies have been shown to have a possible predisposition in African populations.


Complement deficiencies of the terminal cascade proteins also predispose patients to infection, but
the clinical history of these patients is different. The terminal complement proteins are the proteins
in the cascade that form the MAC, ie, complement proteins C5-C9. These proteins are responsible
for bactericidal killing of organisms such as N meningitidis. Class I MHC molecules are expressed
by nearly all nucleated cells of vertebrates. Class I MHC recognition (choice A) is carried out by Tcell receptors that can recognize antigens only in association with the proteins of the major
histocompatibility complex (MHC).

IL-2 (choice B) is a proliferation cytokine produced by the TH cells. TH cell activation is initiated by
interaction of the TCR-CD3 complex with a processed antigenic peptide bound to a class II MHC
molecule on the surface of an antigen-presenting cell.

A 20 month old male infant presents with a history of recurrent bruising, bilateral otitis media,
pneumonia, and severe eczema. Complete blood count with differential count reveals
thrombocytopenia (50 103/mm3), low mean platelet volume (5 fL), low serum IgM, and increased
IgA and IgE levels.

What is the likely diagnosis for this patient?


1)
2)
3)
4)
5)

Ataxia-telangiectasia
DiGeorge syndrome
Hyperimmunoglobulinemia E syndrome
Nezelof syndrome
Wiskott-Aldrich syndrome (WAS)

The correct answer is choice E.

WAS first manifests itself by defective responses to bacterial polysaccharides and by lower-thanaverage IgM levels. Other responses and effector mechanisms are normal in the early stages of the
syndrome. As the WAS sufferer ages, there are recurrent bacterial infections and a gradual loss of
humoral and cellular responses. The syndrome includes thrombocytopenia (lowered platelet count;
the existing platelets are smaller than usual and have a short half-life), which may lead to fatal
bleeding. Eczema (skin rashes) in varying degrees of severity may also occur, usually beginning
around one year of age. The defect in WAS has been mapped to the short arm of the X
chromosome and involves a cytoskeletal glycoprotein present in lymphoid cells called sialophorin
(CD43).The WAS protein is required for assembly of actin filaments required for the formation of
microvesicles.

Ataxia telangiectasia (choice A) is a disease syndrome that includes deficiency of IgA and

sometimes of IgE. The syndrome is characterized by difficulty in maintaining balance (ataxia) and
by the appearance of broken capillaries (telangiectasia) in the eyes. The primary defect appears to
be in a kinase involved in regulation of the cell cycle. The relationship between the immune
deficiency and the other defects in ataxia telangiectasia remains obscure.

In patients with DiGeorge syndrome (choice B), children are born without a thymus and therefore
lack the T-cell component of the cell-mediated immune system. They are generally able to cope
with infections of extracellular bacteria, but cannot effectively eliminate intracellular pathogens.
Although B cells are present in normal numbers, affected individuals do not produce antibody in
response to immunization with specific antigens. The lack of functional cell-mediated immunity
results in repeated infections with viruses, intracellular bacteria, and fungi. Even the attenuated
virus present in a vaccine, capable of only limited growth in normal individuals can produce lifethreatening infections. It is a developmental defect, which is associated with the deletion in the
embryo of a region on chromosome 22. It also manifests along with characteristic facial
abnormalities, hypoparathyroidism, and congenital heart disease.

Hyper-IgE (Job syndrome) (choice C) is a primary immunodeficiency characterized by skin


abcesses, recurrent pneumonia, eczema, and elevated levels of IgE. It is accompanied by facial
abnormalities and bone fragility. This multi-system disorder is autosomal dominant and has variable
expressivity. The gene for hyper IgE syndrome, or HIES, maps to chromosome 4. HIES
immunologic signs include recurrent infection and eosinophilia in addition to elevated IgE levels.
Although other symptoms are similar to WAS, thrombocytopenia is missing here.

Thymic hypoplasia, or the Nezelof syndrome (choice D), is an inherited disorder. The mode of
inheritance for this rare disease is not known and its presentation varies, making it somewhat
difficult to diagnose. As the name implies, thymic hypoplasia is a defect in which a vestigial thymus
is unable to serve its function in T-cell development. In some patients, B cells are normal, whereas
in others a B-cell deficiency is secondary to the T-cell defect. Affected individuals suffer from
chronic diarrhea, viral and fungal infections, and a general failure to thrive. The severity of this Xlinked disorder increases with age and usually results in fatal infection or lymphoid malignancy.
Initially, T and B lymphocytes are present in normal numbers.
A 42-year-old man presents in your office, having suffered with a chronic cough and low fever for
two weeks. Physical examination reveals a temperature of 38.0 C. Chest radiograph reveals a
diffuse bilateral reticulonoduar pattern. A transbronchial biopsy is performed and microscopic
examination shows focal areas of inflammation containing epithelioid macrophages, Langhans giant
cells, and lymphocytes.
These findings are most typical for which of the following immunologic responses?
1)
2)
3)
4)
5)

Graft versus host disease


Polyclonal B-cell activation
Type I hypersensitivity
Type II hypersensitivity
Type IV hypersensitivity

The correct answer is choice E.

Granuloma formation with reactivation or reinfection tuberculosis in an adult is a classic type IV


hypersensitivity reaction. Type IV hypersensitivity is also known as cell mediated or delayed type
hypersensitivity. It is involved in the pathogenesis of many autoimmune and infectious diseases,
such as tuberculosis, leprosy, blastomycosis, histoplasmosis, toxoplasmosis, leishmaniasis; and
granulomas due to infections and foreign antigens. Another form of delayed hypersensitivity is
contact dermatitis (poison ivy, chemicals, heavy metals, etc.) in which the lesions are more papular.

Mechanisms of damage in delayed hypersensitivity include T lymphocytes and monocytes and/or


macrophages. Cytotoxic T cells (Tc) cause direct damage whereas helper T (TH1) cells secrete
cytokines which activate cytotoxic T cells and recruit and activate monocytes and macrophages,
which cause the bulk of the damage. The delayed hypersensitivity lesions mainly contain
monocytes and a few T cells.

Diagnostic tests in vivo include delayed cutaneous reaction (e.g. Montoux test) and patch test (for
contact dermatitis). In vitro tests for delayed hypersensitivity include mitogenic response, lymphocytotoxicity and IL-2 production. Corticosteroids and other immunosuppressive agents are used in
treatment.

Graft versus host disease (choice A) is a rare disorder that occurs when the immune cells from
donated bone marrow stem cells or a blood transfusion attack the recipient. It does not produce a
granulomatous reaction.

Polyclonal B-cell activation (choice B) is a natural mode of immune response in mammals.


Granulomatous reactions are based mostly upon cell-mediated immunity.

Type I hypersensitivity (choice C) is also known as immediate or anaphylactic hypersensitivity. The


reaction usually takes 15 - 30 minutes from the time of exposure to the antigen.

Type II hypersensitivity (choice D) is also known as cytotoxic hypersensitivity and typically affects a
variety of organs and tissues. The reaction time is minutes to hours.

The status of an individual's immune system can be evaluated through the balance of
cytokines produced. This increasingly important classification method is referred to as the Th1/Th2
balance. The cytokines produced, including interleukins and interferons, can be grouped into those
secreted by T-helper1(Th1) type cells and those secreted by T-helper2 (Th2) type cells. In general,
Th1 cells promote cell-mediated immunity while Th2 cells induce humoral immunity.
Which of the following diseases is not known to be Th2 dominant?
1) Bronchial Asthma

2)
3)
4)
5)

Atopic dermatitis
Insulin dependant diabetes
Systemic lupus erythematosus
Ulcerative colitis

The correct answer is choice C

While the characterization is not absolute,T-helper cell cells tend to be CD4 cells while Tsuppressor and regulatory cells tend to be CD8 cells.

There are essentially 4 different types of immune-mediated hypersensitivity disorders. All include an
inflammatory component with varying degrees of tissue/organ damage.

Type 1-Immediate hypersensitivity. Prototypic disorders include atopic bronchial asthma,


anaphylaxis and allergies. Type I reactions include the rroduction of IgE antibody with the
immediate release of vasoactive amines (e.g. histamine) and other mediators (leukotrienes,
Prostaglandin D2 , enzymes, and platelet activating factor (PAF) and pre-inflammatory

cytokines) from mast cells. Type-1 hypersensitivity is a Th2 mediated response.

Type II- Antibody mediated hypersensitivity. Type II hypersensitivity ischaracterized by


opsonization and phagocytosis. Prototypic disorders include auto-immune hemolytic
anemias, Goodpasture Syndrome, Graves' Disease, Myesthenia gravis, Type II diabetes,
pemphigus and acute rheumatoid arthritis. It is Th1 mediated.

Type III-Immune Complex (IC) mediated hypersensitivity. It is Th1 mediated and the
prototypic disorders include Systemic Lupus Erythematosus, serum sickness and some
forms of glomerulonephritis. This type is composed of three phases:

1. Deposition of antigen-antibody complexes

2. Complement activation and the recruitment of inflammatory cells

3. Release of enzymes and toxic metabolites

Type IV-Cell Mediated- hypersensitivity. It is characterized by the release of TH1-type


cytokines and T-cell mediated cytotoxicity. Prototypic diosrders include contact dermatitis,
multiple sclerosis, type I diabetes, rheumatoid arthritis, the inflammatory bowel diseases and
tuberculosis. These types of reactions are carried out by both CD4 and CD8 T cells.

Of the possible answers listed, asthma (choice A), SLE (choice D) and ulcerative colitis (choice
E) are Th2-dominant. Atopic dermatitis (choice B) appears to be the result of the sequential
activation of Th2 followed by Th1 cells. Type I diabetes is a Type IV reaction and is not
characterized by Th1 or Th2 dominance.

Hypersensitivity refers to the excessive, undesirable (damaging, uncomfortable and sometimes


fatal) reactions produced by the immune system. Hypersensitivity reactions require a pre-sensitized
(immune) state of the host and can be divided into four types: type I, type II, type III and type IV,
based on the mechanisms involved and time of onset. Frequently, a particular clinical condition
(disease) may involve more than one type of reaction.
Which of the following is a type II hypersensitivity reaction?
1)
2)
3)
4)
5)

Pemphigus Vulgaris
Graves' disease
Goodpasture's syndrome
Only B and C are Type II reactions
All off the above are Type II reactions

The correct answer is choice E

There are essentially 4 different types of immune-mediated hypersensitivity reactions. All include an
inflammatory component with varying degrees of tissue/organ damage.

Type 1-Immediate hypersensitivity. Prototypic disorders include atopic bronchial asthma,


anaphylaxis and allergies. Type I reactions include the production of IgE antibody with the
immediate release of vasoactive amines (e.g. histamine) and other mediators (leukotrienes,
Prostaglandin D2 , enzymes, and platelet activating factor (PAF) and pre-inflammatory cytokines)
from mast cells. Type-1 hypersensitivity is a Th2 mediated response.

Type II- Antibody mediated hypersensitivity. Type II hypersensitivity is characterized by


opsonization and phagocytosis. Prototypic disorders include auto-immune hemolytic anemias,
Goodpasture Syndrome, Graves' Disease, Myesthenia gravis, Type II diabetes, pemphigus and
acute rheumatoid arthritis. It is TH1 mediated.

Type III-Immune Complex (IC) mediated hypersensitivity. It is TH1 mediated and the prototypic
disorders include Systemic Lupus Erythematosus, serum sickness and some forms of
glomerulonephritis. This type is composed of three phases:

Deposition of antigen-antibody complexes

Complement activation and the recruitment of inflammatory cells

Release of enzymes and toxic metabolites

Type IV-Cell Mediated- hypersensitivity. It is characterized by the release of TH1-type cytokines


and T-cell mediated cytotoxicity. Prototypic diosrders include contact dermatitis, multiple sclerosis,
type I diabetes, rheumatoid arthritis, the inflammatory bowel diseases and tuberculosis. These
types of reactions are carried out by both CD4 and CD8 T cells.

Of the disorders listed, all are Type II, antibody-mediated reactions.

In pemphigus (Choice A), an auto-antibody is directed against desmogleins. These


are cadherins which play a role in forming desmosomes.

In Graves' disease (Choice B), stimulatory auto-antibodies form to the TSH-receptor (TSH-r).

In Goodpasture's syndrome (Choice C), auto-antibodies form to a domain of the alpha-3


chain of collagen typeIV. Goodpasture's syndrome is also known as anti-glomerular
basement membrane disease and is characterized by pulmonary hemorrhage and
glomerulonephritis.

Therefore, the correct answer is Choice E, all of the above are Type II reactions.
Table 1: Summary of Hypersensitivity reactions

Hypersensitivity refers to excessive, undesirable (damaging, discomfort-producing and sometimes


fatal) reactions produced by the normal immune system. Hypersensitivity reactions require a presensitized (immune) state of the host. Hypersensitivity reactions can be divided into four types: type
I, type II, type III and type IV, based on the mechanisms involved and time taken for the reaction.
Frequently, a particular clinical condition (disease) may involve more than one type of reaction.
What type of hypersensitivity is associated with the hyperacute rejection of a transplanted kidney?
1) Type I hypersensitivity reaction
2) Type II hypersensitivity reaction
3) Type III hypersensitivity reaction

4) Type IV hypersensitivity reaction


5) Mixed type (II and III) hypersensitivity reaction

The correct answer is choice B

Hyperacute transplant rejections may occur within minutes of the transplant and are most often
associated with incompatibility with HLA antigens. More recently, MHC-associated antigens have
also been associated with hyperacute reactions. Whether the antibodies are directed against HLA
or MHC loci, the reaction is a Type II, antibody-mediated response (Choice B)

Type II- Antibody mediated hypersensitivity. Type II hypersensitivity is characterized by


opsonization and phagocytosis. Prototypic disorders include hyperacute organ juection, autoimmune hemolytic anemias, Goodpasture Syndrome, Graves' Disease, Myesthenia gravis, Type II
diabetes, pemphigus and acute rheumatoid arthritis. It is TH1 mediated.

Table 1: Summary of hypersensitivity reactions

The induction of self-tolerance in T cells results from exposure of immature thymocytes to self
antigens and the subsequent clonal deletion of those that are self-reactive. Any tissue antigens that
are sequestered from the circulation, and are therefore not seen by the developing T cells in the
thymus, will not induce self-tolerance. Exposure of mature T cells to such normally sequestered
antigens at a later time might result in their activation.
All of the following are a clinical manifestation of the above situation except?
1) Neonatal myasthenia gravis

2)
3)
4)
5)

Autoimmunity after vasectomy


Autoimmunity to Myelin basic protein (MBP) after head injury
Release of lens protein after eye damage
Systemic Lupus Erythematosus

The correct answer is choice A.

As stated above, any self antigen that the developing T cells do not encounter during development
will be treated as non-self; hence the choices B, C and D are correct.

Myelin basic protein (MBP) is an example of an antigen normally sequestered from the immune
system, by the blood-brain barrier. However, trauma to tissues following either an accident or a viral
or bacterial infection can also release sequestered MBP (choice C) into the circulation resulting in
autoimmunity.

Sperm arise late in development and are sequestered from the circulation. However, after a
vasectomy, some sperm antigens are released into the circulation and can induce auto-antibody
formation in some men (choice B).

Similarly, the release of lens protein after eye damage or of heart-muscle antigens after myocardial
infarction has been shown to lead on occasion to the formation of auto-antibodies (choice D).

In patients with myasthenia gravis (choice A) auto-antibodies that bind to the acetylcholine
receptors on the motor end-plates of muscles are produced, blocking the normal binding of
acetylcholine and also inducing complement mediated lysis of the cells. As a result, a progressive
weakening of the skeletal muscles occurs. Ultimately, the antibodies destroy the cells bearing the
receptors. Although this is an autoimmune disorder, acetylcholine receptors are not sequestered in
the body.
A 3 month old male infant was admitted for investigation of repeated infections and failure to thrive.
He was born full term and APGAR scores were appropriate. He was not given BCG at birth. The
infant was healthy till 2 months of age when he was administered antibiotics for a chest infection,
following which he developed diarrhea and re-infection of his chest occurred. Due to this, further
immunizations were postponed. There were no rashes or lymphadenopathy but his liver was
palpable just below the right costal margin. He had slight tachycardia and tachypnoea on
examination. Investigations showed a marked deficiency of T cells with normal numbers of B cells,
with no immunoglobulin production. He had a T- B+ form of severe combined immune deficiency.
What would be the normal course of therapy for the patient?
1) Treatment with IFN-
2) Bone marrow transplant
3) Administration of immunoglobulins

4) Administration of interleukin-12
5) All of the above

The correct answer is choice B.

Severe combined immunodeficiency (SCID) is a rarer immunodeficiency that affects both B and T
cells, if untreated, results in death from infection at an early age. SCID stems from various defects
in lymphoid development that affect either T cells or both T and B cells. All forms of SCID have
common features. Clinically, SCID is characterized by a very low number of circulating
lymphocytes. There is a failure to mount immune responses mediated by T-cells.

Cell replacement therapy for immunodeficiencies is possible by bone-marrow transplantation.


Replacement of stem cells with those from an immunocompetent donor allows development of a
functional immune system. Careful matching of patients with donors and the ability to manipulate
stem-cell populations to select CD34+ precursor cells has minimized the risk in this procedure,
even when no ideal donor exists. T cells are depleted and CD34+ stem cells are enriched before
introducing the donor bone marrow into the SCID infant.

The administration of recombinant IFN- (choice A) is an effective therapy for patients with chronic
granulomatous disease (CGD). Therapy of CGD patients with IFN- significantly reduces the

incidence of infections. Also, the infections that are contracted are less severe.

Administration of immunoglobulins (choice C) is helpful in disorders where antibodies are absent


such as specific IgA and IgG deficiency, common variable immunodeficiency (CVI),
agammaglobulinemia etc.

Recombinant IL-2 (choice D) may help to restore immune function in diseases where immunity
against a virus is required. It is generally used in therapy in AIDS patients and in certain cancers.
An 8-year old boy presents to the emergency department in respiratory distress. His symptoms
began within 15 minutes after he ate a chocolate chip cookie, and he has had similar reactions after
eating foods containing peanuts and other ingredients. Physical examination reveals diffuse
urticaria, angioedema and bilateral wheezing. The patient is treated with epinephrine,
antihistamines and oral corticosteroids, and he recovers.
What type of hypersensitivity reaction has this patient experienced?
1) Type I
2) Type II
3) Type III
4) Type Iva
5) Type Ivb
6) MIxed Type hypersensitivity

The correct answer in choice A

While categorizing the type of allergy, it is essential to note down the history of the patient
especially allergies and the time of the onset of symptoms from the probable antigen exposure. In
the above mentioned case, the history of food allergies and the immediate reaction after the cookie
consumption all shows that it is a Type I hypersensitivity which was caused probably due to
presence of traces of peanut proteins in the cookie. Type I hypersensitivity is an immediate type of
response against an allergen which is mediated through IgE antibodies and mast cells.

Peanut allergy affects around 1% of population in industrial countries and is still growing. Peanuts
and other food related allergies can lead to life threatening allergic reactions. Normally food
antigens like peanut proteins do not provoke immune responses in normal individuals. They neither
mount TH2 type cell response nor cause the production of IgE antibodies. However, atopic
individuals (Atopic individuals are those individuals who genetically produce greater number of IgE
antibodies and therefore are more prone to the allergies) mount TH2 type cell response against
specific proteins/antigens.

Cytokines produced by TH2 cells, stimulate antigen-specific B cells to switch to IgE antibody
secretion. Antigen exposure would result in the secretion of antigen-specific IgE antibodies and

there attachment to the mast cells. This makes mast cells sensitive against a specific antigen such
as a peanut protein and therefore this stage is referred to as sensitization. Further exposure, known
as a rechallenge to antigen would result in the activation of Ig E- coated mast cells and their
simultaneous degranulation and release of mediators. Degranulated mast cells release lipid
mediators, cytokines, amines and proteases and cause tissue damage, vascular dilatation, smooth
muscle contraction and inflammation which are the hallmark of an immediate allergic response.

The most severe form of allergic reaction is called Anaphylaxis, which is caused by widespread
degranulation of mast cells in response to an antigen. During Anaphylaxis, edema of various
tissues occurs including the larynx causing an airway obstruction accompanied by a fall in blood
pressure. Both conditions can be life threatening if left untreated. Current treatment is antihistamine
and epinephrine.

A 25-year-old man presents in your urgent care center with a high fever and cough productive of
yellowish sputum that he has had for approximately 24 hours. Laboratory tests reveal a serum IgA
of 22 mg/dL, IgG of 175 mg/dL, and IgM of 40 mg/dL. Auscultation of his chest reveals a few
crackles in both lung bases. A chest radiograph shows bilateral patchy pulmonary infiltrates.
Which of the following inflammatory cell types will most likely be increased in his sputum specimen?
1) Macrophage
2) Mast cell

3) Neutrophil
4) Lymphocyte
5) Langhans giant cell

The correct answer is choice C.

To diagnose pneumonia, health care providers rely on a patient's symptoms and findings from
physical examination. Information from a chest X-ray, blood tests, and sputum cultures are helpful
to confirm the diagnosis. Bacterial infections like pneumonia promote an acute inflammatory
response dominated by neutrophils, which gives the sputum the yellowish, purulent appearance.

In addition to a cough that produces greenish or yellow sputum, people with infectious pneumonia
often have and a high fever that may be accompanied by shaking chills. Shortness of breath is also
common, as is pleuritic chest pain, a sharp or stabbing pain, either felt or worse during deep
breaths or coughs. The decreased serum immunoglobulins at his age suggest common variable
immunodeficiency, which puts him at risk for bacterial infections such as pneumonia.

The role of macrophages (choice A) is to phagocytose cellular debris and pathogens either as

stationary or as mobile cells and are mainly part of the process of resolution of inflammation.

Mast cells (choice B) degranulate and release vasoactive substances as part of type 1
hypersensitivity reactions.

Lymphocytes (choice D) are the 'round cells' of chronic inflammatory infiltrates.

Langhans giant cells (choice E) are typically seen with infectious granulomas.

A 47-year-old man presents in your office with a variety of complaints. Over the past year he has
developed pain and swelling around the small joints of his hands and feet. Additionally, he has
chronic diarrhea and has lost significant weight - approximately 3.5 kg. His chart reveals that the
patient has had several upper respiratory tract infections during this time, including such organisms
as Staphylococcus aureus, Streptococcus pneumoniae, and Pseudomonas aeruginosa cultured. A
stool specimen is found to have Giardia lamblia cysts. Laboratory studies show a hemoglobin of 8.8
g/dL with a reticulocyte count of 3.2%. His serum urea nitrogen is 19 mg/dL with creatinine 1.0
mg/dL. His antinuclear antibody test is negative. Quantitative serum immunoglobulins show IgA 70
mg/dL, IgG 303 mg/dL, and IgM 64 mg/dL. By flow cytometry there are normal numbers of T cells

and slightly reduced numbers of B cells.


Which of the following conditions is most likely to be present in this man?
1)
2)
3)
4)
5)

Bruton agammaglobulinemia
Common variable immunodeficiency
Polymyositis
HIV infection
Diabetes mellitus

The correct answer is choice B.

The patient has common variable immunodeficiency (CVID), with hypogammaglobulinemia that
predisposes him to bacterial infections and also Giardia infection. Patients with CVID can also
develop rheumatoid arthritis and hemolytic anemia.

CVID can involve the following: (1) low levels of most or all of the immunoglobulin classes, (2) a
lack of B lymphocytes or plasma cells that are capable of producing antibodies, and (3) frequent
bacterial infections. A diagnosis of CVID is reserved for those with an undefined B-cell dysfunction.
CVID is diverse, both in its clinical presentation and in the types of deficiency. Although decreased

serum levels of IgG and IgA are characteristic, approximately 50% of patients with the deficiency
also have diminished serum IgM levels and T-lymphocyte dysfunction.

Patients come to medical attention due to infectious diseases at the time of onset, the most
common being otitis media, diarrhea, pneumonia, and sinusitis. Almost all have acute and recurrent
infections. Persistent diarrhea and malabsorption caused by Giardia lamblia infection also occur in
patients with CVID, leading to significant weight loss and, in children, failure to thrive.

The mainstay of treatment for CVID is Ig replacement therapy with antimicrobial therapy initiated at
the first sign of infection. Although expensive, Ig replacement therapy stops the cycle of recurrent
infections.

Bruton's disease (choice A) manifests early in childhood with a marked reduction in all serum
globulins.

Polymyositis (choice C) is characterized by the presence of Anti Jo antibodies in >65% of patients.


Elevated serum creatine kinase is also characteristic, but not specific to polymyositis.

The bacterial infections suggest hypogammaglobulinemia, not a primary or secondary T cell defect
as could occur with HIV infection (choice D).

Though persons with diabetes mellitus (choice E) are prone to bacterial infections, they not likely to
get a Giardia diarrhea or hemolytic anemia in conjunction.

A 38-year-old man arrives in the emergency room following a motor vehicle accident with multiple
lacerations to the lower left leg. He has significant blood loss, requiring transfusion of blood
products. In taking his oral medical history, it is revealed that he has frequent bouts of diarrhea of
unknown etiology and has had a significant number of respiratory tract infections over the past ten
years. During the transfusion, he has an anaphylactic transfusion reaction.
Which of the following diseases is he most likely to have?
1)
2)
3)
4)
5)

Selective IgA deficiency


Graft versus host disease
System lupus erythematosus
Niemann-Pick disease
Amaloidosis

The correct answer is choice A.

Approximately one out of every 600 people has selective IgA deficiency. Among those with this
disease, people of European ancestry greatly outnumber those of other ethnic groups. People with
this deficiency lack IgA, a type of antibody that protects against infections of the mucous
membranes lining the mouth, airways, and digestive tract. IgA deficiency is caused by faulty B cells.
While patients have normal numbers of B cells, these cells do not mature into normal IgA-producing
cells. Scientists do not know the exact cause or causes for these immature B cells. Sometimes
clusters of cases occur in families. People with IgA-deficiency are more likely than the general
population to be related to someone with combined variable immunodeficiency, another form of
immune deficiency.

Many people with IgA-deficiency are healthy, with no more than the usual number of infections.
Those who do have symptoms typically have recurring ear, sinus, or lung infections that may not
respond to regular treatment with antibiotics. People with IgA-deficiency are likely to have other
problems, such as allergies, asthma, chronic diarrhea, and autoimmune diseases.

IgA deficiency is diagnosed by doing tests to measure the amount of total Ig in the blood as well as
IgG2. There is no specific treatment for selective IgA deficiency. Bacterial infections are treated with

antibiotics.

Graft versus host disease (choice B) is rare, but possible from a blood transfusion, however the
results take weeks to manifest.

In systemic lupus erythematosus (choice C), there is more likely to be a polyclonal gammopathy.
There is no tendency to type 1 hypersensitivity reactions.

Niemann-Pick disease (choice D) is a metabolic disease that is typically fatal by 18 months. There
is an adult form of Niemann-Pick disease, but immunologic reactions are not part of this disease.

Amyloidosis (choice E) occurs when amyloid proteins build up in your organs and is not marked by
hypersensitivity reactions.

A 77-year-old woman was admitted because of sudden onset of massive angioedema of her
tongue associated with laryngeal stridor. She was treated with intravenous hydrocortisone. This
was her fifth such episode. She had also experienced an anaphylactoid attack two months earlier.

She had no history of previous allergy and no family history of atopy. The patient was hypertensive
since two years and was taking oral frusemide and captopril for treatment. Captopril was
discontinued and her mild hypertension was managed with alternative medication. The attacks
never recurred.
Which of the following allergic reactions was responsible for the above mentioned symptoms in the
patient?
1)
2)
3)
4)
5)

Type I
Type II
Type III
Type IV
Delayed-type hypersensitivity reaction

The correct answer is choice A

Captopril is an angiotensin-converting enzyme (ACE) inhibitor and this group of drugs is known to
cause severe episodes of angioedema. ACE inhibitors block ACE, the enzyme that among other
actions, degrades bradykinin. Interference with the degradation of bradykinin, a potent vasoactive
nonapeptide is responsible for ACE inhibitor induced angioedema. Most cases are observed in the
first week after starting the medicine; however, up to 30% may occur months or even years after

starting the medicine.

Angioedema is the swelling of deep dermis, subcutaneous or submucosal tissue due to vascular
leakage. Clinically, angioedema is usually nonpitting and nonpruritic sometimes with mild pain.
Laryngeal swelling however can be life-threatening.

The variable portion of the immunoglobulin is represented by which of the following:


1) The antigenic end of the heavy and light chains

2)
3)
4)
5)

The antigenic ends of the light chains


The constant end
A and B
All of the above

The correct answer is choice A

B cells harbor receptors termed as the B cell receptors (BCR) which enable B cells to bind and
respond to epitopes (the part of the antigen recognized by BCR) in presence of specific signals.
Activation of B cells through their BCRs leads to the differentiation of B cells into plasma cells which
are capable of producing soluble forms of the BCR, commonly known as Antibodies. Unlike the T
cell receptors which remain attached to the T cells, soluble antibodies generated from B cells are
capable of performing their functions throughout the whole body away from their site of production.

Antibodies are glycoproteins. The antibody molecule is a Y-shaped structure consisting of four
chains, two light and two heavy chains. Both of the chains contain one Variable (V) and one or
more Constant (C) regions. The first amino acids present at the amino terminal of antibodies, varies
in all antibodies. Therefore these are known as the Variable regions. No two B cells are likely to
produce the antibodies with the same variable regions.

This variability is most pronounced in the region known as the Hypervariable region. Three
hypervariable regions of the light and heavy chains form the Antigen binding site where an epitope
can be recognized and attached. These are also known as the Complementarity determining
regions. The fragment of antibody consisting of the variable (V) and constant domains of the light
and heavy chain are required to bind to the antigens and are therefore known as the Fab (fragment
antigen binding).

A 32 year old G3P2 pregnant (22 weeks) woman presented to the local hospital case room and
complained about her fetus not growing. The patient had an uneventful first pregnancy and had a
normal 5 year old son. Her second pregnancy ended with dead hydropic fetus at 24 weeks. The
fetus had a very low hemoglobin (70g/l), indicating hemolytic disease of the newborn. The woman
was Rhesus (Rh) negative while the fetus was found to be Rhesus D positive. Rh alloimmunization
was suspected.
Which antibody subtype is responsible for Rh alloimmunization?
1)
2)
3)
4)
5)

IgA
IgG
IgM
IgD
IgE

The correct answer is choice B

Hemolytic disease of the newborn (HND) or Erythroblastosis fetalis is caused when blood antigen
incompatibility occurs between the fetus and the mother, usually due to Rh alloimmunization.
During pregnancy, fetal red blood cells are separated from the mothers circulation by the
trophoblast. During her first pregnancy with an Rh+ fetus, an Rh woman is usually not exposed to
enough fetal red blood cells to activate her Rh-specific B cells. At the time of delivery, however,
separation of the placenta from the uterine wall allows larger amounts of fetal umbilical-cord blood
to enter the mothers circulation. These fetal red blood cells activate Rh-specific B cells, resulting in
production of Rh-specific plasma cells and memory B cells in the mother. The secreted IgM
antibody clears the Rh+ fetal red cells from the mothers circulation, but the memory cells remain.
Activation of these memory cells in a subsequent pregnancy with an Rh+ fetus results in the
formation of IgG anti-Rh antibodies, which cross the placenta and damage the fetal red blood
cells.This fetal disease ranges from mild to very severe, and fetal death from heart failure (hydrops
fetalis) can occur.

Pregnancies at risk of HND are those in which an Rh D-negative mother becomes pregnant with an
RhD-positive child (the child having inherited the D antigen from the father). HDN can also be
caused by an incompatibility of the ABO blood group. It arises when a mother with blood type O
becomes pregnant with a fetus with a different blood type (type A, B, or AB). The mother's serum
contains naturally occurring anti-A and anti-B, which tend to be of the IgG class and can therefore
cross the placenta and hemolyse fetal RBCs.

HDN due to ABO incompatibility is usually less severe than Rh incompatibility because fetal RBCs
express less of the ABO blood group antigens compared with adult levels. In addition, in contrast to
the Rh antigens, the ABO blood group antigens are expressed by a variety of fetal (and adult)
tissues, reducing the chances of anti-A and anti-B binding their target antigens on the fetal RBCs.
Only IgG antibodies can cross the placenta. IgA antibodies (choice A) are involved in the mucosal
immunity and do not cross the placenta.

IgM and IgD antibodies (choice C and D respectively) are expressed on the surface of B cells in the
early stages of B cell mediated (humoral) immunity, are also unable to cross the placenta.

IgE (choice E) antibodies cause degranulation of mast cells to release histamines, and are
implicated in type 1 hypersensitivity reactions. IgE immunoglobulins cannot cross the placental
barrier.

Hypersensitivity refers to undesirable (damaging, discomfort producing and sometimes fatal)


reactions produced by the normal immune system. Hypersensitivity reactions require a presensitized (immune) state of the host. Hypersensitivity reactions can be divided into four types: type
I, type II, type III and type IV, based on the mechanisms involved and time taken for the reaction.
Frequently, a particular clinical condition (disease) may involve more than one type of reaction.

An appropriate, useful type I hypersensitivity response of the immune system, accompanied by


eosinophilia, would most likely be directed against which of the following?
1) Flavivirus
2) E. coli O157
3) Spirochetes
4) Liver flukes
5) Staph. Aureus

The correct answer is choice D

Liver flukes are a type of parasitic worm or helminth, a multi-cellular organism that infects both
humans and animals. While infection frequently cases morbidity and occasionally may be fatal,
helminthes have evolved highly complex, immunomodulatory mechanisms that allow them to
coexist with their host.

The immune response to parasitic infections has been termed skewed Th2. Infection is followed
by eosinophilia and production of IgE. The specific immune response is determined by the location
of the infection. When the parasites have infected a lumen, for example, the gut, type 1
hyerpsensitivity and antibody dependent cellular cytotoxic (ADCC) mechanisms block the larvaes
migration into the gut and initiate elimination of the parasite. Muscle contractions and mucus

secretion in the gut expel adult helminthes from the GI system.

Tissue infections are primarily mediated by larvae. Antibodies, including IgE, IgG, and IgA, bind to
the infecting organism and trigger destruction by cells that have receptors to the Fc portion of the
antibody. In some cases, the resulting immune reaction culminates in the development of a
granuloma which surrounds and isolates the parasite, stopping its migration through the tissue.

Epidemiologic studies have shown an inverse relationship between parasitic infections and the
prevalence of allergy and autoimmune disease. These conclusions may seem counter-intuitive.
Allergic symptoms are triggered by the IgE-mediated degranulation of mast cells and basophils and
the consequent release of histamine. Nevertheless, experimental data have shown that parasitic
infections can inhibit allergic reactions and that treatment of parasitic infections can result in
increased cutaneous response to allergens.

Immune responses to viral or bacterial infections are primarily mediated by IgG.

Type I Hypersensitivity Review


It is also known as immediate or anaphylactic hypersensitivity. The reaction may involve skin

(urticaria and eczema), eyes (conjunctivitis), nasopharynx (rhinorrhea, rhinitis), bronchopulmonary


tissues (asthma) and gastrointestinal tract (gastroenteritis). The reaction may cause from minor
inconvenience to death. The reaction takes 15-30 minutes from the time of exposure to the antigen.
Sometimes the reaction may have a delayed onset (10-12 hours).

Immediate hypersensitivity is mediated by IgE. The primary cellular component in this


hypersensitivity is mast cell or basophil. The reaction is amplified and/or modified by platelets,
neutrophils and eosinophils. A biopsy of the reaction site demonstrates mainly mast cells and
eosinophils. The mechanism of reaction involves preferential production of IgE, in response to
certain antigens, and allergens. IgE has very high affinity for its receptor on mast cells and
basophils. A subsequent exposure to the same allergen cross links the cell-bound IgE and triggers
the release of various pharmacologically active substances. Cross-linking of IgE Fc-receptor is
important in mast cell triggering. Mast cell degranulation is preceded by increased Ca++ influx,
which is a crucial process; ionophores which increase cytoplasmic Ca++ also promote
degranulation, whereas, agents which deplete cytoplasmic Ca++ suppress degranulation.

The reaction is amplified by PAF (platelet activation factor) which causes platelet aggregation and
release of histamine, heparin and vasoactive amines. Eosinophil chemotactic factor of anaphylaxis
(ECF-A) and neutrophil chemotactic factors attract eosinophils and neutrophils, respectively, which
release various hydrolytic enzymes that cause necrosis. Eosinophil may also control the local
reaction by releasing arylsulphatase, histaminase, phospholipase-D and prostaglandin-E, although
this role of eosinophils is now in question.

Diagnostic tests for immediate hypersensitivity include skin (prick and intradermal) tests,
measurement of total IgE and specific IgE antibodies against the suspected allergens. Total IgE
and specific IgE antibodies are measured by a modification of enzyme immunoassay (ELISA).
Increased IgE levels are indicative of atopic condition, although IgE may be elevated in some non
atopic diseases (e.g., myelomas, helminthic infection, etc.).

Symptomatic treatment is achieved with antihistamines which block histamine receptors.


Chromolyn sodium inhibits mast cell degranulation, probably, by inhibiting Ca++ influx. Late onset
allergic symptoms, particularly bronchoconstriction which is mediated by leukotrienes are treated
with leukotriene receptor blockers (Singulair, Accolate) or inhibitors of cyclooxygenase pathway
(Zileutoin). Symptomatic, although short term relief from bronchoconstriction is provided by
bronchodilators (inhalants) such as isoproterenol derivatives (Terbutaline, Albuterol). Thophylline
elevates cAMP by inhibiting cAMP-phosphodiesterase and inhibits intracellular Ca++ release is also
used to relieve bronchopulmonary symptoms.

There appears to be a genetic predisposition for atopic diseases and there is evidence for HLA (A2)
association.

Hyposensitization (immunotherapy or desensitization) is another treatment modality which is

successful in a number of allergies, particularly to insect venoms and, to some extent, pollens. The
mechanism is not clear, but there is a correlation between appearance of IgG (blocking) antibodies
and relief from symptoms. Suppressor T cells that specifically inhibit IgE antibodies may play a role.

Table 1: Comparison of Different Types of hypersensitivity

Cytokines are any of a number of small proteins that are secreted by specific cells of the immune
system and glial cells, which carry signals locally between cells, and thus have an effect on other
cells. They are a category of signaling molecules that are used extensively in cellular
communication. They are proteins, peptides, or glycoproteins. The term cytokine encompasses a
large and diverse family of polypeptide regulators that are produced widely throughout the body by
cells of diverse embryological origin.
Which of the following is a cytokine produced by T-cells that acts mainly to activate B-cells to
proliferate:
1)
2)
3)
4)
5)

Interleukin 1
Interleukin 2
Interleukin 4
Interleukin 10
Interferon gamma

The correct answer is choice B

The functions of the cytokines in immune regulation are complex and still in the process of being
defined. Many if not all of the cytokines have multiple functions and many are secreted by more

than one cell type.

IL2 is an exception to the latter; it is produced by Th1 cells and acts to stimulate both T and B cells
and to induce a number of cytokines and adhesion molecules.

IL1 is made by B cells, dendritic cells, macrophages, and other immune cells. It acts across a broad
spectrum, stimulating T and B cells and resulting in production of a number of cytokines.

IL4 is produced by a number of types of T cells as well as mast cells. It induces Th2 cells and,
consequently, isotype switching to IgE.

IL10 is made by T and B cells, macrophages, and monocytes and acts to inhibit T cell proliferation
and IL2 secretion. It down-regulates MHC II expression and also stimulates B cell differentiation.

Interferon gamma is primarily antiviral. It is made by T cells and natural killer (NK) cells. In addition
to inhibiting viral replication, it increases expression of MHC I and also MHC II. It activates

macrophages and down-regulates the Th2 response.

A 4-year-old boy presents to his primary care physician for a well-child examination and routine
immunization. In the time between his first measles-mumps-rubella (MMR) vaccine and now, he
has developed severe atopic dermatitis. Evaluation by an allergist/immunologist revealed IgEmediated allergy to egg, soy, and peanut. He has had anaphylaxis after eating peanuts, and eating
eggs results in urticaria and worsening of his atopic dermatitis. His mother read that MMR vaccine
is produced from eggs and is concerned about a possible reaction.
What is the next course of action for this patient?
1) Proceed with the vaccination
2) Refer the patient for desensitization to egg protein and then proceed with

immunization
3) Delay vaccination because of the history of documented egg allergy
4) Repeat skin prick testing and, if negative, proceed with immunization
5) Use gradually increasing doses of the vaccine to decrease the risk of a reaction

The correct answer is choice A

The instances of food allergy are most frequent in childhood and mostly such conditions are
improved with age. MMR is a triple viral vaccination for measles, mumps and rubella. The

vaccination of measles and mumps are the live attenuated viruses which are grown in fibroblasts
derived from the chicken embryo while attenuated rubella virus is obtained from diploid human
fibroblasts. Despite of all the controversies regarding MMR being the cause of egg allergy, the Food
Allergy Committee of Spanish Society of Clinical Immunology and Pediatric Allergy has declared
MMR vaccination to be safe for children with egg allergy except with the patients with severe
anaphylactic reactions after egg ingestion are recommended to receive vaccination in their
reference hospital.

According to one report, MMR vaccine contains only negligible amount of egg proteins (0.5 to 1 ng
of ovoalbumin per 0.5 ml dose of vaccine) while compared to this amount there is a higher amount
of gelatin (14.5 mg) and neomycin sulfate (25ug) contained in a single dose of vaccine and
therefore these allergens are the thought to be the likely candidates for the anaphylactic reactions
after the MMR vaccination.

A 54-year-old male was diagnosed with IgG1 multiple myeloma. The patient had rapidly
progressive refractory disease with the serum paraprotein rising 610 g/l each month, having
progressed following multiple previous chemotherapeutic regimes and autologous stem cell
transplantation. Finally immunotherapeutic strategies were used to treat this advanced refractory
myeloma with a series of four vaccinations using autologous idiotype-protein pulsed dendritic cells
combined with adjuvant GM-CSF. The vaccinations were well tolerated with a mild fever postvaccination. An idiotype-specific T-cell proliferative response developed which was associated with
the production of gamma-interferon, indicating a TH-1-like response. The patient also developed

anti-idiotype IgM antibodies although no idiotype-specific cytotoxic T-cell response could be


demonstrated.
Which one of the following statements is false regarding this immunotherapeutic strategy used in
this case?
1) Idiotypes are created by the hypervariable regions on the Fab fragment of IG's
2) The epitopes that are shared within a species are called idiotypic determinants
3) The idiotype of the immunoglobulin produced by myeloma cells may function as a

tumour-specific antigen
4) Dendritic cells are powerful antigen-presenting cells, pivotal in initiating immune

response
5) Vaccination in cancer patients with tumour-antigen loaded dendritic cells may result in

clinical response

The correct answer is choice B.

The epitopes that are shared within a species and differ between species are called isotypic
determinants (not idiotypic determinants) . Differences in antibodies among individuals of same
species are called the allotypic determinants. Isotypic and allotypic determinants are associated
with the constant regions of heavy and light chain genes of immunoglobulin (Ig). Hence option B is

incorrect.

Idiotypic determinants confer an immunoglobulin (antibody) molecule an antigenic individuality that


is analogous to the individuality of the molecule's antibody activity. Unique antigenic determinants
are present on individual antibody molecules. Idiotypes are the antigenic determinants created by
the hypervariable regions on the Fab fragment of the Ig molecules (option A)

The malignant cells in myeloma rearrange their immunoglobulin genes and produce a monoclonal
immunoglobulin which is secreted into the serum. This immunoglobulin is unique to the malignant
clone and presents antigenic determinants, or idiotypes, within their variable regions which may
function as a tumour-specific antigen (option C). Therefore these idiotypes represent a potential
target for immunotherapy utilizing these proteins to generate an anti-idiotype immune response as
in this case.

Dendritic cells (DC) are powerful antigen-presenting cells which are pivotal in initiating the immune
response (option D). They are capable of activating naive T lymphocytes through the presentation
of antigenic peptides bound to major histocompatibility complexes (MHC) in the presence of
appropriate co-stimulatory signals. Recently, vaccination of patients with tumour-antigen loaded DC
has resulted in clinical responses in B-cell lymphoma and melanoma (option E)

A 7 month old female infant presents to your office with a complaint of chronic cough for the past 3
months and post-tussive vomiting for past 3 weeks. The infant was born full-term with no neonatal
complications. There is no history of asthma or recurrent infection in the family. Physical
examination reveals a small, emaciated infant (weight at < 5th percentile, height at 10th percentile),
extensive oral thrush, generalized lymphadenopathy (including cervical, axillary, inguinal, and
suboccipital), bilateral intercostal and subcostal retraction, bilateral crackles, hepatosplenomegaly,
and extensive diaper rash. Serum IgG, IgA, and IgM levels are all above normal.
What is the most likely diagnosis for this patient?
1)
2)
3)
4)
5)

Adhesion molecule deficiency


Asthma
Brutons agammaglobulinemia
HIV infection
Severe combined immunodeficiency (SCID)

The correct answer is choice D.

Presence of opportunistic infections may point to an immunodeficiency. Failure to thrive,


generalized lymphadenopathy and high immunoglobulin levels could be an indication of HIV
infection.

The classic descriptions of LAD (choice A) (leukocyte adhesion deficiency) includes recurrent
bacterial infections, defects in neutrophil adhesion, and a delay in umbilical cord sloughing. The
defects in adhesion result in poor neutrophil chemotaxis and phagocytosis. A WBC differential will
reveal extremely elevated levels of neutrophils (of the order of 6-10x normal) because they are
unable to leave the blood vessels. Specific diagnosis is made through monoclonal antibody testing
for CR3, one of the three complete proteins which fail to form properly as a result of -2 integrin
subunit deficiency.

Asthma patients (choice B) show high IgE levels, high eosinophils, along with bronchospasm and
respiratory distress all of which are absent in this case.

X-linked agammaglobulinaemia (Bruton's disease) is characterized by


panhypogammaglobulinaemia with absent antibody production and the absence of mature B
lymphocytes. Since, in this case, the antibody titres are all above normal, it cannot be Brutons
disease (X-linked agammaglobulinaemia) (choice C).

Severe combined immune deficiency, (SCID) (choice E) is characterized by defects in either B or T


cells or both with no immunoglobulin production. Thus, although patients with SCID suffer from

recurrent infections, their immunoglobulin levels are not high as in this case.
A 22 year old caucasian woman presents to your outpatient clinic with a 7 day history of a sinus
infection. When questioned, she reveals a history of recurrent, respiratory infections and allergy
symptoms that occur regularly each spring. She is otherwise healthy and does not take any regular
medication except birth control pills. She has no known drug allergies but does complain of
environmental allergies. In terms of family history, her younger sister has been diagnosed with
juvenile rheumatoid arthritis.
Which of the following can be suspected pending completion of laboratory analyses?
1) IgG deficiency
2) IgA deficiency
3) Deficiency of antibodies to polysaccharide antigens
4) IgG3 deficiency
5) IgM deficiency

The correct answer is choice B.

IgA is the most common antibody in the body and is found both in serum and in the secretions of
the respiratory, gastrointestinal and genitourinary systems.

Specific deficiency of IgA is the most common primary immunodeficiency. The prevalence of IgA
deficiency is influenced by ethnicity; studies in US Caucasians have shown prevalence that varies
between 1:233 to 1:3,000. Asians have a lower prevalence of the condition with studies showing
prevalence of 1:2,600 to 1:18,500. Prevalence among individuals in the Arabian peninsula has
been shown to be 1:143. Transmission of the deficiency is autosomal dominant with incomplete
penetrance.

Many IgA-deficient individuals exhibit no symptoms. However, some IgA-deficient patients


experience recurrent infections, primarily sinopulmonary and gastrointestinal, as well as allergies
and autoimmune disease. Recurrent sinopulmonary infections are the most common condition. GI
conditions experienced by the patient deficient in IgA include infections, malabsorption, lactose
intolerance, celiac disease, and ulcerative colitis. It is thought that these patients impaired mucosal
clearance may predispose them or act as a causative factor in their developing food and inhalant
allergies. Studies on the relative prevalence of allergy among IgA deficient patients vary with most
finding a higher risk of allergy in this population. Individuals deficient in IgA have an elevated risk of
developing a range of autoimmune disorders; further, the risk of autoimmune disease is elevated in
first-degree relatives of these patients.

Because IgA-deficient patients may develop antibodies to IgA, blood transfusions are a critical
concern. Most sources recommend that patients wear a medic-alert bracelet and that they receive
washed red cells to avoid a possible anaphylactic reaction. Similarly, patients should avoid IVIG;
they do not need the IgG and residual IgA in the formulation may trigger an immune reaction.

Management of the patient is focused on treating the infections or other conditions they experience.

Because IgA is abundant in respiratory secretions, its absence predisposes individuals to repiratory
infections. Reduced levels of IgG or IgM would not be reflected in specific respiratory infections.

A 34 year old woman was admitted to the hospital emergency room feeling unwell, with nausea and
vomiting. She had lost 12 kg over the preceding year and her BMI was much below normal. She
also complained of feeling anxious, with dizziness on standing, worsening fatigue and increased
generalized pigmentation over her chest and face. The patient had completed 6 weeks of
counseling for anxiety prior to hospital admission but no improvement was noted. BP lying was
94/58 mmHg and standing 86/55 mmHg (normal 120/80) with a low volume pulse 74/min. There
were patches of vitiligo on the elbows and ankles. Investigation findings are summarized below.

Based on the lab findings, what should be the appropriate treatment for this patient?
1) Thyroxine
2) Hydrocortisone
3) Insulin
4) IV fluids
5) Doxepin

The correct answer is choice B.

In this case, a diagnosis of Addison's disease was made and the treatment administered was IV
hydrocortisone, oral fluocortisone and IV fluids. The sodium on Day 5 was 138 mmol/l. Six weeks
later the patient reported improvement in her general health while on oral hydrocortisone and
fludrocortisone.

Addison's disease is a rare disease. The clinical features and treatment of the condition are well
established. However, the diagnosis is still often delayed or missed with potentially serious
consequences. It is not uncommon for these patients to be misdiagnosed with other conditions
such as chronic fatigue, thyroid disease (often co-existent), depression and rheumatological
conditions.

Addisons disease is an autoimmune adrenal failure that can occur early in life as part of
autoimmune polyendocrine syndrome type 1 (APS type 1) due to a mutation in the autoimmune
suppressor gene (AIRE) located on chromosome 21q22.3, or in adults in isolation or as genetically
more complex autoimmune polyendocrine syndrome type 2 (APS type 2). About 80% of patients
with APS type 1 develop Addison's disease while about 18% develop type 1 diabetes mellitus.
Adrenocortical antibodies (ACA) can be found in between 60-80% patients or antibodies against 21

hydroxylase (21OHAb) can be found in high proportion (8090%) of patients with Addison's disease
at diagnosis. Because aldosterone is absent, hyponatremia, with low chloride and hyperkalemia are
often present. Hyponatremia is the most common finding and occurs in 90% of patients.
Hyperkalemia is found in 60-70% of patients. For this, reason IV fluids are administered along with
hydrocortisone, to balance the electrolytes. However, this alone cannot form the treatment regime
(Choice D).

As explained in the table, introduced in 1960s the short synacthen test remains the gold standard
test for the biochemical diagnosis of Addison's disease. Cortrosyn is a synthetic corticotropin, which
is intravenously administered with a dose of 350 mg. Serum cortisol levels are measured from
blood samples drawn after 30 and 60 minutes. Peak serum cortisol levels greater than 635 nmol/l
exclude the diagnosis of adrenal insufficiency because the response to stimulation is considered
adequate at this level. Cortisol levels of 500-635 nmol/l are indeterminate. Cortisol levels of less
than 500 nmol/l suggest adrenal insufficiency.

Chronic fatigue, dizziness and anxiety along with others are common features with thyroid
imbalance (choice A) and most commonly misdiagnosed in people with Addisons disease.
Doxepin (choice E) is a psychotropic agent with tricyclic antidepressant and anxiolytic properties
generally used for patients suffering from depression.

Diabetes is often co-existent with Addisons disease. The symptoms typically associated with
Addisons disease are due absence of adrenal hormones and hence insulin is administered only if
diabetes is also observed.
Hypersensitivity refers to excessive, undesirable (damaging, discomfort-producing and sometimes
fatal) reactions produced by the normal immune system. Hypersensitivity reactions require a presensitized (immune) state of the host. Hypersensitivity reactions can be divided into four types: type
I, type II, type III and type IV, based on the mechanisms involved and time taken for the reaction.
Frequently, a particular clinical condition (disease) may involve more than one type of reaction.
Which type of hypersensitivity reaction does the Mantoux test measure?
1) Type I
2) Type II
3) Type III
4) Type IV
5) Mixed type of reaction (III and IV)

The correct answer is choice D

The Mantoux test is a skin test used to test for exposure to Mycobacterium tuberculosis. It is also
known as the PPD ( Purified Protein Derivative) test. It is a skin test which, if positive, reveals
previous exposure to the M. tuberculosis antigen. The test itself is performed by injecting a small
quantity of protein antigen subcutaneously and measuring the reaction ~48 hours later. False
positives may occur if the patient has been vaccinated with the BCG vaccine, is being treated with

immunosuppressive agents, has had a booster TB vaccination or has had TB.

Interpretation:

1. Negative test: little to no swelling

2. Positive test:

A patient with a compromised immune system (e.g. HIV +):> or = 5 mm

Patients at risk for tuberculosis: > or = 10 mm

Patients at low risk: > or = 15mm

Table 1: Review of Hypersensitivity Reactions

A 30 year old man presented to your outpatient clinic complaining of weight loss, night sweats, low
grade fever and general malaise. Examination revealed bilateral cervical and axillary

lympadenopathy but hepatomegaly and splenomegaly were however, absent. White cell count was
normal, hemoglobin was low and erythrocyte sedimentation rate (ESR) was 75mm/h (normal value:
male < 10 mm/h). Histological examination of cervical lymph nodes revealed the presence of large
binucleate cells. Bone marrow cells were normal and involvement of other lymph nodes was not
seen.
What is the likely diagnosis for this patient?
1)
2)
3)
4)
5)

Stage I Hodgkins lymphoma


Stage II Hodgkins lymphoma A
Stage II Hodgkins lymphoma B
Stage I Non-Hodgkins lymphoma
Burkitts lymphoma

The correct answer is choice C.

The large binucleate cells called Reed-Sternberg cells are characteristic feature of the Hodgkins
disease or Hodgkins lymphoma. This patient is diagnosed as having a stage 2 Hodgkin's disease,
because, although only lymphoid tissue above the diaphragm is involved (since spleen and liver are
not enlarged), his ESR is above 40mm/h. Also since lymphoma is no longer asymptomatic, but the
patient experiences some symptoms, the suffix 'B' is added to the stage which suggests a poorer
prognosis associated with systemic symptoms.

The Ann Arbor staging scheme is a common method for classification of Hodgkins lymphoma.
According to this classification, stage I (choice A) is involvement of a single lymph node region most
commonly the cervical region or a single extralymphatic site. Stage II involves two or more lymph
node regions on the same side of the diaphragm or of one lymph node region and a contiguous
extralymphatic site. The stage III Hodgkins lymphoma involves lymph node regions on both sides
of the diaphragm, which may include the spleen and/or limited contiguous extralymphatic organ or
site. In the stage IV is disseminated involvement of one or more extralymphatic organs. The
absence of systemic symptoms is signified by adding 'A' to the stage; the presence of systemic
symptoms is signified by adding 'B' to the stage. For localized extranodal extension from mass of
nodes that does not advance the stage, subscript 'E' is added.
Thus, a stage II Hodgkins lymphoma A (choice B) would involve two or more lymph organs without
development of any symptoms.

Most lymphomas that are not Hodgkins type are classified as of non-Hodgkins type (absence of
Reed-Sternberg cells) and include about 10 different types of disease (choice D).
The translocation of cmyc leads to constitutive expression of c-Myc and an aggressive, highly
proliferative B-cell lymphoma called Burkitts lymphoma (choice E). Burkitts lymphoma can be
diagnosed by t(8;14) c-myc translocation.

Figure 1: Summary of Ann Arbor staging for Lymphoma

An 18-year-old student with end-stage renal failure due to chronic glomerulonephritis was given a
cadaveric kidney transplant. He had been on maintenance haemodialysis for 2 months, and on
antihypertensive therapy for several years. His tissue type was HLA-A1, -A9, -B8, -B40, -Cw1, Cw3, -DR3, -DR7. The donor kidney was matched for blood group one DR antigen and four of six
ABC antigens. He was given triple immunosuppressive therapy with cyclosporin A, azathioprine
and prednisolone.He passed 5 litres of urine on the second postoperative day and his urea and
creatinine fell appreciably.
However, on the seventh postoperative day, his graft became slightly tender, his serum creatinine
increased and he had a mild pyrexia. A clinical diagnosis of acute rejection was confirmed by a
finding of lymphocytic infiltration of the renal cortex on fine-needle aspiration. A 3-day course of
intravenous methylprednisolone was started. Twenty-four hours later his creatinine had fallen and
urine volume increased.
Which one of the following sets of antigens do not require a histocompatibility testing for a
successful renal grafting?
1)
2)
3)
4)
5)

MHC class II antigens


Lewis red cell antigens
MHC class I antigens
ABO red cell antigens
Rho antigens

The correct answer is choice E.

Rho (D) antigens are not expressed on endothelial tissue and therefore play no apparent role in
graft rejection or survival. An organ from a donor with ABO type B positive can be safely
transplanted into a recipient with ABO type B negative.

Histocompatibility testing

"Histo" means "tissue." Histocompatibility is defined as a measure of how well 2 tissues "get along
with one another" when they find themselves in a confined space. Histocompatibility testing is used
to minimize graft foreignness and reduce donor-specific immune responses to the transplanted
organ. The ABO and HLA systems have been identified as the major transplantation antigens in
man.

Major histocompatibilitycomplex proteins, or MHC proteins serve as carriers of antigenic material on


the surface of APCs. In humans, the MHC gene loci is divided into three regions: Class I, II, and III.
In humans, MHC genes are referred to as human leukocyte antigen (HLA) genes. There are three
different important class I MHC genes in humans and they are called HLA-A, B and C. HLA-D

genes belong to MHC class II which include HLA-DP, DQ, and DR. When 2 people share the same
HLAs, they are said to "match." In other words, their tissues are immunologically compatible with
each other. Typing for these MHC class I antigens (choice C) as well as for MHC class II antigens
(choice A) is important in determining compatibility for organ transplantation. Many studies have
shown a stepwise decrease in graft survival of cadaver kidneys with increasing numbers of HLA
mismatch. The superior results with HLA-A, HLA-B, and HLA-DR mismatches have led to a system
of mandatory sharing of such donor kidneys.

At least 15 different minor red cell antigen systems have been identified in humans. The most
important of these appears to be the Lewis system. (choice B). Transplant recipients who are highly
sensitized to minor red cell antigens as a result of numerous blood transfusions, for example, may
experience antibody-mediated rejection responses (hyperacute or chronic rejection). For this
reason, the potential recipient's blood is screened for the presence of antibodies to the known
minor red cell antigens before transplantation.

ABO typing (choice D) detects ABO compatibility. Basic ABO compatibility depends on the
presence or absence of antigens on donor RBCs and the presence or absence of specific
antibodies to these antigens in the recipient's serum. Anti-ABO antibodies are of the IgM
classification and cause agglutination, complement fixation, and hemolysis. If an ABO-incompatible
graft is transplanted, hyperacute rejection will occur. In kidney transplantation, preformed circulating
cytotoxic antibodies in the recipient react with ABO isoagglutinins produced by the graft, and the
graft quickly turns dark and soft as a result of diffuse thrombosis of the microvasculature.

A 76-year-old man presents in your office with a 4-month history of oculodynia, blurred vision, and
weight loss of 4.5 kg. He had no fever, chills, or diaphoresis. A complete blood count revealed
leukocytosis with 98% lymphocytes. His past medical history and family history were unremarkable.
Physical examination revealed a visual field defect peripherally. He had splenomegaly but no
adenopathy, hepatomegaly, or skin lesions. Additional blood studies showed a positive direct
Coombs, but haptoglobin and bilirubin were normal. A bone marrow biopsy was performed. Flow
cytometry was performed on the peripheral blood and bone marrow aspirate for
immunophenotyping. The lymphocytes were positive for IgM and IgD and were negative for CD3,
CD4 and CD8.
Select the correct answer for the type of cells present.

1)
2)
3)
4)
5)

A mature, MHC class I restricted T-cell


A mature, MHC class II restricted T-cell
A mature B-cell
An immature T-cell
An immature B-cell

The correct answer is choice C

B cell maturation proceeds in antigen-independent and antigen-dependent stages. Antigenindependent maturation includes development from precursor cells in the marrow through the naive
B cell (a cell that has not been exposed to antigen previously) found in the peripheral lymphoid
tissues. Antigen-dependent maturation occurs following the interaction of antigen with naive B cells.
Once a B cell encounters its cognate antigen and receives an additional signal from a T helper cell,
it can further differentiate into plasma B cells and memory B cells. Plasma cells are terminally
differentiated B cells responsible for synthesis and secretion of immunoglobulin. The final products
of B cell development are circulating long-lived memory B cells and plasma cells found
predominantly in primary follicles and germinal centers of the lymph nodes and spleen.

Each B cell has a unique receptor protein (referred to as the B cell receptor (BCR)) which is a
membrane-bound immunoglobulin on its surface that will bind to one particular antigen.
Immunoglobulins (Ig) or antibodies (Ab) are -globulins that are produced by plasma cells in
response to an immunogen. Igs occur in two forms: a soluble form secreted into the blood and
other fluids and a membrane-bound form that is attached to the surface of a B cell. The membranebound form of an antibody is part of the BCR, which allows a B cell to detect a specific antigen and
triggers B cell activation. The BCR is composed of surface-bound IgD or IgM antibodies.

The majority of B cells express both IgM and IgD on the surface and are derived from pre-B cells
found mainly in the bone marrow. Immature B cells express IgM (choice E) while mature B cells
express both IgM and IgD on their surface (choice C). Pre-B cells contain intracytoplasmic IgM but
do not express surface immunoglobulin. Early B cell precursor is TdT+, CD34+, HLA-DR+, then
undergoes heavy (H) chain rearrangement and adds CD19, then adds CD10, then adds IgM heavy
chain; then adds light (L) chain rearrangement and adds cytoplasmic IgM with heavy and light
chains, then B cells express IgM and IgD with the same binding site, then adds CD20 (now called
preB cell); then adds surface Ig, then adds CD21 and CD22 and drops TdT (now called B cell).

If B cell encounters an antigen that interacts with its variable region, it becomes a plasma cell

Differentiation antigens
Different populations of lymphocytes have different surface proteins called differentiation antigens,
i.e. different cell types produce different and sometimes unique surface proteins that can serve as
an identifying marker for that type of cell. For example, all the cells that express the Thy-1 protein
are T cells; all T cells have it, and no B cells or NK cells do. Some T cells have the cluster of
differentiation marker #4 = CD4 and others have CD8. CD4 cells are largely helper T cells while
CD8 cells are largely cytotoxic T cells( refer to table).

B cells have been commonly identified by other surface markers in addition to immunoglobulins.
These include the receptor for the Fc portion of immunoglobulins, B cell-specific antigens CD19 and
CD20, and surface antigens coded for by the HLA-D genetic region in humans. All mature B cells
bear surface immunoglobulin that is the antigen-specific receptor. The major role of B cells is
differentiation to antibody-secreting plasma cells. However, B cells may also release cytokines and
function as antigen-presenting cells.

CD4-, CD8- and CD4+, CD8+( choice D) are immature T cells. Mature cytotoxic T-cells are CD4-,
CD8+ and MHC class I restricted (choice A) while mature helper T-cells are CD4+, CD8- and MHC
class II restricted (choice B). T-cell receptor (TCR) complex contains an accessory molecule, called
CD3, that is essential of signal transduction. All T cells are positive for CD3.

A 4 year old boy presents to his primary care physician for routine examination and immunization.
In the time between his first measles-mumps-rubella (MMR) vaccine and now, he has developed
severe atopic dermatitis. Evaluation by an allergist/immunologist revealed IgE-mediated allergy to
egg, soy, and peanuts. He has had anaphylaxis after eating peanuts, and eating eggs results in
urticaria and worsening of his atopic dermatitis. His mother read that MMR vaccine is produced
from eggs and is concerned about a possible reaction.

What is the next course of action for this patient?


1)
2)
3)
4)
5)

Proceed with the vaccination


Repeat skin prick testing and, if negative, proceed with immunization
Use gradually increasing doses of the vaccine to decrease the risk of a reaction
Delay vaccination because of the history of documented egg allergy
Refer the patient for desensitization to egg protein and then proceed with
immunization

The correct answer is choice A.

Approximately 0.5 percent of children have an allergy to eggs. Well-designed studies of more than
1200 children with severe allergic reactions to eggs have clearly shown that these children can
safely receive MMR vaccine. Although the MMR vaccine is produced in chicken embryo cell culture,
this material does not contain significant amounts of egg white cross-reacting (ovalbumin) proteins.
The Red Book Committee concluded that children with egg allergy are at low risk for anaphylactic
reactions to MMR.

In 1994, the American Academy of Pediatrics recommended that children with allergies to chicken,
eggs or the feathers not receive the MMR until skin testing could be performed (choice B).
However, because skin testing has not been shown to be predictive of reactions to MMR vaccine, it

is no longer recommended before administering MMR vaccine.

The 1994 guidelines (no longer followed) also stated that in allergic children vaccine should be
given in six doses with each dose having increasing amounts of vaccine in it (choice C).

People with allergies to chickens or feathers have never been shown to be at increased risk for
reaction to the vaccine so delaying the vaccine (choice D) or desensitization to egg protein (choice
E), which is not very easy, is not advised.

A 50-year-old man with end stage renal disease from long-standing diabetes mellitus receives a
cadaveric renal transplant. A month later, he has increasing malaise. His urine output decreases.
Laboratory studies show a rising serum urea nitrogen and creatinine. The allograft is biopsied and
seen microscopically to be undergoing destruction by cells which are recognizing graft cells
expressing class I HLA antigens.
Which of the following cells is most likely to mediate acute cellular rejection of solid organ
transplants?
1) B cells
2) Eosinophils

3) Basophils
4) T cells
5) Dendritic cells

The correct answer is choice D

Acute cellular rejection is mediated by the T cells. CD4 and CD8 cells recognize the allo- (foreign)
antigens on the graft and trigger carefully orchestrated and complex web of changes in the
cytokines (e.g., IL2), enzymes (e.g., hydrolysis of inositol phospholipid), minerals (e.g., calcium),
and other actors that regulate the cellular immune response. Cytotoxic T cells (CTL) are activated
and release mediators that directly target the foreign cells through a variety of mechanisms.

Experience with allograft programs and research into the biology of graft rejection has identified
strategies for minimizing the immunogenicity of the graft. For example, brain death in the donor has
been shown to result in hemodynamic instability and to cause release of cytokines and other
mediators in the organ to be transplanted; these changes put the organ into a proinflammatory
state. Recent research has shown that treating the donor, after death, with substances such as
catecholemines can interfere with development of this proinflammatory state in the donor organ
and can improve graft survival.

B cells that are the foundation of humoral (antibody) immunity can play a role in rejection but
humoral rejection is differentiated from acute cellular rejection both by the mechanism (B cell vs. T
cell) and also by the clinical consequences. Humoral immunity is responsible for hyperacute
rejection that occurs in the first minutes to hours following transplantation. Eosinphils primary role is
to combat parasitic and viral infections; they also infiltrate the lungs of asthmatic patients. Basophils
also play a role in parasitic infections as well as in allergy. Dendritic cells are found in the surfaces
of the body (skin, gut) and act as antigen presenting cells.

Acute cellular rejection is the most common form of rejection. It develops generally within the first
month after surgery but it may appear at any time, even years after immunologic quiescence.
Clinical signs are fever, graft tenderness and pain, oliguria and rapid functional deterioration with
proteinuria. With the modern immunosuppressive regimens acute rejection has become less
frequent and generally less severe so that most of the aforementioned clinical signs may be absent.
The rapid increase of serum creatinine level (>25% of the basal value) remains the most common
sign. A T cell response to donor HLA antigens is considered to have a major role in the
pathogenesis of this form of rejection.

A 27-year-old woman is given intravenous penicillin to treat infective endocarditis. Within minutes of
starting this therapy, she begins to have severe difficulty breathing with respiratory stridor and
tachypnea. She suddenly develops an erythematous skin rash over most of her body.

Her symptoms are most likely to be produced by what type of immunological phenomenom?
1)
2)
3)
4)
5)
6)

Type I
Type II
Complement-mediated reaction
Type IV
IgA-mediated reaction
This is a non-immunological reaction

The correct answer is choice A

Adverse drug reactions are classified as predictable (Type A), typically exaggerations of the drugs
known effects and dose-dependent, or unpredictable (Type B), which are not related to the drugs
pharmacology. IgE-mediated allergic reactions, also called Type 1 reactions, to penicillin are a Type
B reaction. If anaphylactic, these reactions can be life-threatening.

Prediction of severe penicillin allergy is difficult. While the presence of IgE antibodies reacting with
the metabolites of penicillin are necessary for a severe allergic reaction, many individuals with
detectable IgE antibodies have no adverse clinical reaction to the drug. Penicillin skin testing is,
according to one reference, about 60% predictive of clinical hypersensitivity.

True IgE-mediated drug allergies are rare. Several studies have shown that only a small fraction of
patients evaluated for allergy after experiencing an adverse reaction to penicillin had confirmed
penicillin allergy. True IgE-mediated reactions are expected to become more severe with time.

Type 2 reactions, cell-mediated responses, are rare and would manifest more slowly. A Type 4
reaction would be expected to manifest as contact dermatitis and is most frequently experienced by
healthcare workers who have frequent exposure to penicillin.

Table 1. Comparison of Different Types of hypersensitivity reactions

A 3 year old girl was referred to the hospital by her pediatrician for immunological tests. She
suffered from repeated infections and had had several episodes of otitis media and pneumonia.
Immunological tests revealed the following information.

IgM 440 mg dl1 (normal 40240 mg dl1)


IgG 50 mg dl1 (normal 6391344 mg dl1)
IgA levels undetectable (normal 70312 mg dl1)

Further analysis of blood lymphocytes was advised. Flow cytometric analysis revealed normal
expression of CD40 and CD40L on B cells. However, cDNA analysis revealed a defective
activation-induced cytidine deaminase (AID) gene. The diagnosis of autosomal AID deficiency
hyper IgM syndrome was made and intravenous immunoglobulin (IVIG) therapy was started, which
resulted in a dramatic decrease in the frequency of infections.
Which of the following statements is not true for patients with AID deficiency?
1)
2)
3)
4)
5)

It is also a hyper IgM syndrome caused by a mutation in AID gene


Patients are susceptible to opportunistic infections
It can occur due to defect in the uracil-DNA glycosylase (UNG) gene
Patients have enlarged lymph nodes in a streptococcal infection
It is a defect due to absence of immunoglobulin class switching

The correct answer is choice B.

In X-linked hyper IgM, the absence of class switching is due to defect in the CD40 Ligand that
prevents its interaction of CD40. Interaction between CD40 on the B cell and CD40L on the T
Helper cell is required for B-cell activation and the absence of this co-stimulatory signal inhibits the
B cell response to T-dependent antigens. Generally, patients with X-linked hyper IgM syndrome
show extensive B-cell defects and thus, susceptibility to both pyogenic and opportunistic infections.

Hyper IgM which results from defects in the AID gene or the gene encoding uracil-DNA glycosylase
(UNG) (choice C) and resembles X-linked agammaglobulinemia in that these patients have
increased susceptibility to pyogenic infections only and not to opportunistic infections such as
Pneumocystis carinii and Cryptosporidium, which are characteristic of the X-linked hyper IgM
syndrome.

The enzyme activation-induced cytidine deaminase (AID) converts cytidine to uridine. It triggers a
DNA repair mechanism that induces isotype switching. B cells that lack functional AID are unable to
generate somatic mutations after activation and therefore cannot undergo affinity maturation. The
only response these B cells can make to the activating signals is to proliferate. This results in the
accumulation of IgM+ B cells in the lymphoid organs. The B-cell response to T-independent
antigens, however is unaffected by this defect, accounting for the production of IgM (choice A),
giving rise to an enlarged spleen (splenomegaly) and enlarged lymph nodes (lymphadenopathy)
(choice D). AID deficiency causes absence of isotype switching (choice E).

A 16 year old girl is brought to hospital by her frantic parents after a bee sting. Vital signs are BP,
70/40; RR 30 and laboured; HR 140; T 37.5oC.

Which of the following would not be an appropriate option in her management?


1)
2)
3)
4)
5)
6)

Epinephrine
Diphenhydramine
Methylprednisolone
salbutomol/albuterol
atropine
All of the above are appropriate measures

The correct answer is choice E

The hypotension, tachycardia, fever and hyperventilation following a bee sting indicate an
anaphylactic (type 1) reaction. Atropine is an anti-cholinergic drug and is contraindicated. Adverse
effects of atropine include tachycardia, temperature effects and arrhythmias.

Anaphylaxis is a severe allergic reaction that is rapid in onset and may be fatal. Prominent dermal
and systemic signs and symptoms are usually present. The full-blown syndrome may consist
of urticaria (hives) and/or angioedema along with hypotension and bronchospasm. The classic form
was described in 1902 and involves the prior sensitization of an individual to an allergen. Later reexposure produces symptoms via an immunologic mechanism.

Anaphylaxis also may be idiopathic or be induced by drugs, insect stings and foods (most
commonly peanuts and shellfish). The most common inciting drugs/agents in anaphylaxis are
parenteral antibiotics, especially the penicillins) and IV contrast materials. Other oral and IV
medications and many other types of exposures also have been implicated.

A patient experiencing an anaphylactic reaction may present initially with signs of a respiratory
involvement including sneezing,coughing and complete airway obstruction (secondary to laryngeal
and epiglottic edema). The patient may also complain of tightness in the throat and chest. Voice
changes and stridor may also be evident. Auscultation of the lungs may reveal wheezes and
significant respiratory distress due to bronchospasms of the lower airways. Increased mucous
secretions, caused by the actions of histamine and leukotriene, may also be heard.

Cardiovascular manifestations of anaphylactic reactions will range from mild hypotension to total
vascular collapse and profound shock. Tachycardia will be present as the body attempts to
compensate for the hypotension. Depending upon the degree of shock, peripheral pulses may be
weak or absent.

Due to the sudden onset of symptoms and the multiple organ involvement, quick identification and
treatment are critical. It is imperative that healthcare professionals are able to recognize and

understand the signs and symptoms of an anaphylactic reaction and therefore respond with the
appropriate interventions. Initial patient care measures are directed at providing adequate airway,
ventilatory and circulatory support (ABCs) followed by appropriate treatment.

The drug of choice in an anaphylactic reaction is epinephrine, a sympathomimetic. (Choice A) (0.30.5 mg/kg of a 10-3solution).

Diphenhydramine (Choice B) , an H1 blocker is also an option. (25-50mg , q 6hr)

Methylprednisone, (Choice C) is a steroid and steroids have not been shown to be effective in an
acute situation.

Salbutamol/albuterol (Choice D) is the drug of choice for persistant bronchospasm (10mg/hr or 2.5
mg q 15 minutes).

Table 1: Review of Hypersensitivity reactions

Figure 1: Anaphylaxis algorithm

According to current theory, the most likely pathologic mechanism of insulin-dependent (type I)
diabetes mellitus is:
1) chromosomal translocation
2) an enzyme defect in the red blood cells
3) autoimmune destruction of beta cell in the pancreas
4) high carbohydrate diet over a long period of time
5) chronic emotional stress
6) autoimmune destruction of alpha cell in the pancreas

The correct answer is Choice C

Type 1 diabetes is the most common severe chronic autoimmune endocrine disease and accounts
for 90% of childhood-onset diabetes. It also comprises between 5% to 10% of adult diabetes. In
the US, there are approximately 30,000 new cases diagnosed annually with 1 in 300 children and 1
in 100 adults affected. In the US and around the world, the incidence of type 1 diabetes is
increasing at approximately 2% to 5% per year. Furthermore, the median age of diagnosis is
decreasing. Between 10 and 20 million people affected worldwide with significant social and
economic costs.

Type I diabetes, also known as Insulin Dependent Diabetes Mellitus (IDDM) is an autoimmune
disorder where the insulin producing cells of the pancreas are destroyed in a T-cell

mediated process. There is a genetic linkage with an MHC class II locus (on chromosome 6p21) as
well as weaker linkages with other loci. No chormosomal translocation (Choice A) nor red blood
cell enzyme defect (Choice B) has been implicated to date in the pathogenesis of IDDM.

The primary auto-antigen in IDDM is likely to be insulin, though some studies have implicated
glutamic acid decarboxylase-65 (GAD65), the original 64K antigen and other studies have
implicated pancreatic islet-specific glucose-6-phosphatase catalytic subunit related protein (IGRP)
as a secondary antigens. The differences may simply reflect autoantibody cross-reactivity and not
distinct antigenicity.

A long-term high carbohydrate diet (Choice D) is more important in the pathogenesis of Type II
Diabetes (also known as Non-Insulin Dependent Diabetes Mellitus or NIDDM).

While chronic, long term stress (Choice E) has been shown to be immunosuppressive, it has not
been directly implicated in the pathogenesis of IDDM.

Standard therapy at diagnosis of IDDM involves the replacement of insulin to


prevent life-threatening ketoacidosis and dietary control. Future directions for treatment include use

of hsp60 (Diapep277) and Glutamic acid decarboxylase 65 (GAD65) as competing antigenic


epitopes with the goal of shifting the T-cell-mediated response to a humoral response; the use of
monoclonal antibodies to CD3 and CD20 (Rituximab); and pancreatic cell therapy.

A 36 year-old female presents in your office with persistant dull pain over epigastric area for 2
months with no associated symptoms such as heartburn, nausea, and vomiting. Abdominal
sonography showed a large hepatic mass at left lobe of liver. Initial laboratory data were only
notable for a decreased hemoglobin level of 9.7 g/dL and a mildly elevated level of alkaline
phosphatase (139 U/L). The serum marker of hepatitis B or hepatitis C was also negative. Tumor
markers, including serum carcinoembryonic antigen (CEA), CA19-9, and -fetoprotein level (AFP)
were within normal value. She received booked for an urgent excision of the mass. The excised
liver microscopically showed giant cells, fibrous bands and rare mitotic figures. These tumor cells
were positive for expression of CD21 and CD35 by immunohistochemical staining and positive for
expression of EBER by in situ hybridization. Therefore, the final pathology was follicular dendritic
cell tumor of the liver.
Which of the following statements is false regarding dendritic cells?
1)
2)
3)
4)

They are the most potent of all the antigen-presenting cells


Dendritic cells cannot activate naive T cells
Dendritic cells may develop from a myeloid or lymphoid lineage
IL-12 production by dendritic cells directs the primary immune response along a Thelper1 pathway

5) Langerhans cells are dendritic cells found in the epidermis of the skin

The correct answer is choice B.

Only professional antigen-presenting cells (macrophages, B lymphocytes, and dendritic cells) are
able to activate a resting helper T-cell when the matching antigen is presented. However,
macrophages and B cells can only activate memory T cells whereas dendritic cells can activate
both memory and naive T cells, and are the most potent of all the antigen-presenting cells. Hence
choice B is not a true statement.

Dendritic cells (DCs), are the most potent of all the antigen-presenting cells and represent the
pacemakers of the immune response (choice A). They are crucial to the presentation of peptides
and proteins to T and B lymphocytes and are widely recognized as the key antigen presenting cells
(APCs). They are critical for the induction of T cell responses resulting in cell-mediated immunity
(CMI). The T cell receptors on T lymphocytes recognize fragments of antigens bound to molecules
of the major histocompatibility complex (MHC) on the surfaces of APCs. The peptide binding
proteins are of two types, MHC class I and II, which interact with and stimulate cytotoxic T
lymphocytes (CTLs) and T helper cells (Ths), respectively.

DCs may develop from a myeloid or lymphoid lineage (choice C). The myeloid pathway of
differentiation gives rise to DCs that home to peripheral tissues to take up and process exogenous
Ags prior to migrating to the secondary lymphoid tissues to present Ags to nave T cells. Lymphoid
(thymic) DCs, on the other hand, promote negative selection in the thymus. They are involved in the
presentation of self-Ag to developing thymocytes and, hence, the subsequent deletion of
autoreactive T cells. Precursors of thymic DCs to migrate to the thymus in an immature form and
undergo development exposed only to self-Ag within the thymus.

Dendritic cells are a major source of many cytokines, namely, interferon-alpha (IFN-a), IL-1, IL-6,
IL-7, IL-12, IL-15 etc., all of which are important in the elicitation of a primary immune response.
Interleukin-12 production is critical for the promotion of an effective cellular immune response by
activating and differentiating T lymphocyte to the Th 1 pathway (choice D). Its secretion appears to
be inhibited by various tumour-derived substances, including nitric oxide (NO), prostaglandin E2
(PGE2).

Langerhans cells are dendritic cells in the epidermis, containing large granules called Birbeck
granules (choice E). They are normally present in lymph nodes and other organs, including the
stratum spinosum layer of the epidermis. They can be found elsewhere, particularly in association
with the condition langerhans cells histiocytosis (a rare disease involving clonal proliferation of
langerhans cells related to other forms of abnormal proliferation of WBCs, such as leukemias and
lymphomas).

A 7-year-old boy presents in your office with a sore throat. A throat culture grows group A hemolytic
streptococcus. He receives antibiotic therapy. However, 17 days later he develops dark-coloured
urine. Laboratory studies show 3+ blood on urinalysis. A renal biopsy is performed. On
immunofluorescence staining the biopsy shows granular deposition of IgG and complement around
glomerular capillary loops.
Which of the following is most likely responsible for this pattern of findings?
1)
2)
3)
4)
5)

Type I hypersensitivity
Type II hypersensitivity
Type III hypersensitivity
Type IV hypersensitivity
Host versus graft disease

The correct answer is choice C.

The child has acute proliferative glomerulonephritis, a complication of streptococcal pharyngitis,


which is typical of immune complex deposition, a form of type III hypersensitivity. Type III
hypersensitivity reactions may be general or may involve individual organs including skin, kidneys,
lungs, blood vessels, joints, or other organs. This reaction may be the pathogenic mechanism of

diseases caused by many microorganisms

A type III hypersensitivity reaction is mediated by soluble immune complexes. They are mostly of
the IgG class, although IgM may also be involved. The antigen may be exogenous (chronic
bacterial, viral or parasitic infections), or endogenous (non-organ specific autoimmunity). The
antigen is soluble and not attached to the organ involved. Primary components are soluble immune
complexes and complement (C3a, 4a and 5a). The damage is caused by platelets and neutrophils
and lesions contain primarily neutrophils and deposits of immune complexes and complement.
Macrophages infiltrating in later stages may be involved in the healing process.

The affinity of antibody and size of immune complexes are important in production of disease and
determining the tissue involved. Diagnosis involves examination of tissue biopsies for deposits of
immunoglobulin and complement by immunofluorescence microscopy. The immunofluorescent
staining in type III hypersensitivity is granular (as opposed to linear in type II). The presence of
immune complexes in serum and depletion in the level of complement are also diagnostic.
Polyethylene glycol-mediated turbidity (nephelometry) binding of C1q and Raji cell test are utilized
to detect immune complexes. Treatment includes anti-inflammatory agents.

Type I hypersensitivity (choice A) is characterized by an immediate anaphylactic response.

Type II hypersensitivity (choice B), also known as cytotoxic hypersensitivity, is not characterized by
immune complex deposition.

Type IV hypersensitivity (choice D), also known as cell mediated or delayed type hypersensitivity, is
not typical for cell mediated immunity.

Graft versus host disease (choice E) is a rare disorder that occurs when the immune cells from
donated bone marrow stem cells or a blood transfusion attack the recipient. It does not
characterized by immune complex deposition.

A 4 year old boy was sent to your clinic for detailed immunological examination. The boy was a
normal term delivery with no complications with proper prenatal care. At one month of age, a
staphylococcal abcess on his chest wall had to be treated surgically along with a course of penicillin
antibiotic. In total, he has developed staphylococcal abscess 5 times before the age of one year.
His development status was below the third percentile. Examination showed a tendency to be
pyrexial, with bilateral axillary and inguinal lymphadenopathy and marked hepatosplenomegaly.
Family history revealed the following facts:

The child had 2 elder sisters, who were healthy; two male siblings had died of infections at 8
months and 2 years. Immunological investigation of showed mild anemia (Hb 105g/l; normal (135175 g/L)), IgG and IgA immunoglobulin subclasses showed gross polyclonal elevation. Neutrophils
showed impaired killing of staphylococci. The polymorphs failed to consume oxygen or to produce
hydrogen peroxide during phagocytosis. Thus, a diagnosis of chronic granulomatous disease
(CGD) was made.
Chronic granulomatous disease is characterized by which of the following traits?
1)
2)
3)
4)
5)

X-linked agammaglobulinemia
Defective T-cells and platelets
Absence of class II MHC molecules
Defect in the oxidative pathway
Hypersensitivity reaction

The correct answer is choice D.

CGD is a genetic disease with two distinct forms; an X-linked form that is seen in about 70% of
patients and an autosomal recessive form found in the remainder. This disease is caused due to a
defect in the oxidative pathway by which phagocytes generate hydrogen peroxide and the resulting
reactive products, for e.g. hypochlorous acid, that kill phagocytosed bacteria. CGD patients
undergo excessive inflammatory reactions that may result in gingivitis, swollen lymph nodes, and

nonmalignant granulomas (lumpy subcutaneous cell masses); they are also susceptible to bacterial
and fungal infection. Several related defects may also lead to CGD; these include a missing or
defective cytochrome (cyt b558) that functions in an oxidative pathway and defects in proteins
(phagocyte oxidases, or phox) that stabilize the cytochrome. Also, the general defect in the killer
function of phagocytes, there is also a decrease in the ability of mononuclear cells to serve as
APCs.

Both processing and presentation of antigen are impaired in this condition. Increased amounts of
antigen are required to trigger T-cell help when mononuclear cells from CGD patients are used as
APCs. X-linked agammaglobulinemia (XLA) (choice A) or Brutons hypogammaglobulinemia is a Bcell defect characterized by extremely low IgG levels (not seen in this patient) and by the absence
of other immunoglobulin classes. This is because individuals with XLA have no peripheral B cells.

Defective t-cells and platelets are seen in the Wiskott-Aldrich syndrome (choice B) and involves a
specific defect in the cytoskeletal glycoprotein present in lymphoid cells called sialophorin
(CD43).This protein is required for assembly of actin filaments required for the formation of
microvesicles. Also, low IgM and platelets are observed.

Absence of class II MHC molecules (choice C) causes the bare lymphocyte defect. It is also a type
of immunodeficiency. Without these molecules, the patients lymphocytes cannot participate in
cellular interactions with T helper cells.

Hypersensitivity (choice E) reactions are caused by exaggerated immune responses such as


overproduction of cytokines etc. Hypersensitivity reactions do not involve defects in the oxidative
pathway.
The basic immunoglobulin molecule has a four-chain structure, comprising of two identical heavy
(H) chains and two identical light (L) chains, linked together by inter-chain disulfide bonds. The
aminoterminal portions of the heavy and light chains, characterized by a highly variable amino-acid
composition, are referred to a VH and VL, respectively. The constant parts of the light chain are
designated as CL, while the constant parts of the heavy chains are further divided into three distinct
subunits: CH1, CH2 and CH3. The light chains are bound to the heavy chains by many noncovalent
interactions and by disulfide linkages.
The function of the Variable region of the light chain (CL) includes which of the following:
1)
2)
3)
4)
5)

Variation between different immunoglobulin isotypes


Formation of the complementarity determining region
Felicitating antigen binding by attaching to mast cells
Holding the heavy and light chains together
Allowing cross-linking of antigens

The correct choice is choice B.

The first ~110 amino acids of the amino-terminal region of a light or heavy chain vary greatly
among antibodies of different specificity. These segments of highly variable sequence are called
Variable (V) regions. All the differences in specificity exhibited by different antibodies can be traced
to differences in the amino acid sequences of V regions. Areas of the V regions called
complementarity determining regions (CDRs), and it is these CDRs, on both light and heavy chains,
that constitute the antigen binding site of the antibody molecule.

By contrast, within the same antibody class, far fewer differences are seen when one compares
sequences throughout the rest of the molecule. It is important to note here that the CDR is present
on both the heavy and light chains. The variable regions of the heavy and light chains pair in each
arm of the Y of an immunoglobulin molecule to generate two identical antigen-binding sites, which
lie at the tips of the arms of the Y. It is due to this variability that a large repertoire of antibodies,
capable of binding to different antigens is produced.

The constant region of the heavy chain (CH) is responsible for formation of the different isotypes of
an immunoglobulin molecule (choice A).

The constant regions (CH and CL) interact to hold the molecule together and are involved in
several biological activities, the so-called effector functions such as complement binding,

attachment to mast cells, placental passage and binding to cell membranes (choice C).

The heavy and light chains are bound together (choice D) by many noncovalent interactions and by
disulfide bonds.

The Variable regions of the heavy and light chains pair in each arm of the Y shaped
immunoglobulin molecule to generate two identical antigen-binding sites, which lie at the tips of the
arms of the Y. The possession of two antigen-binding sites allows antibody molecules to cross-link
antigens and to bind them much more stably (choice E)

A 65-year-old male presented complaining of increasing fatigue, exertional dyspnea and ankle
swelling. On examination, he was very pale with signs of congestive cardiac failure. A complete
blood count (CBC) gave the following information

White cell count: 3.7 x 109/l (normal 4-11 x 109/l)


Platelet count: 31 x 109/l (normal 150-400 x 109/l)
Hemoglobin count: 54g/l (normal 135-175 g/l)
Abnormal macrocytic erythrocyte morphology was reported.

For further diagnosis, the following lab tests were carried out:

Serum vitamin B12: 42ng/l (normal 170-900)


Folate: 10.3 ng/ml (normal 7-36 nmol/l)
Total bilirubin: 2.5 mg/dl(normal 0.2-1.9 mg/dL)
Alanine aminotransferase: 35 U/mL (normal 8-35 U/l)
Aspartate aminotransferase: 30 U/mL (normal 8-36 U/l).

What further testing should be done to obtain a definitive diagnosis?


1)
2)
3)
4)
5)

Anti-intrinsic factor antibody screening


Bone marrow examination
Hepatitis A screening
Electrocardiogram
Schilling test

The correct answer is choice A.

Preliminary tests show that there is a vitamin B12 deficiency. The patient was admitted with signs of
a moderate hemolytic anemia. His CBC shows that he has a macrocytic anemia. Leukocyte and
platelet counts are on the lower end of the normal range, indicating a developing pancytopenia. His
increased bilirubin values support a diagnosis of hemolysis. Based on his serum vitamin B12 and

folate results, he can definitively be diagnosed as having a megaloblastic anemia due to a vitamin
B12 deficiency. There is no signs or investigations suggestive of liver disease. Since the liver
enzymes are normal, liver disease is ruled out as a source of jaundice or the macrocytic anemia.
Thus, further testing should be screening for antibodies against intrinsic factor. A common cause of
vitamin B12 deficiency in a patient with no previous gastrointestinal history is pernicious anemia.

Loss of gastric parietal cells due to autoimmunity leads to a lack of intrinsic factor. This leads to
inability to absorb vitamin B12 causing pernicious anemia. Positive testing for antibodies against
intrinsic factor will lead to a diagnosis of pernicious anemia. If Intrinsic factor blocking antibodies are
not found in the serum, the vitamin B12 deficiency is due to some other malabsorption disorder.

A bone marrow examination (choice B) would reveal megaloblastic anemia, caused due to B12
deficiency, but would still not diagnose the cause.

Hepatitis A (choice C) screening is not recommended here since liver functions are all normal (ALT,
AST levels).

The patient shows signs of congestive heart failure due to anemia. An electrocardiogram (ECG)
(choice D) would be recommended if serum B12 levels were normal.

A schilling test (choice E) involves 4 stages testing with radioactive cobalamin with/without intrinsic
factor and it is important to note that it was once widely used for diagnosis of pernicious anemia but
no longer popular because of availability of easier tests such as antibody screening for intrinsic
factor and gastric parietal cells. Hence, anti-intrinsic factor antibody screening is the preferred test
in this case.

John, a 2 month old child, was referred to your specialty clinic for a work-up for his failure to thrive
and his recent development of seizures. A chest x-ray and echocardiogram revealed an enlarged
heart and truncus arteriosus with a large ventricular septal defect. DiGeorge syndrome was
suspected. Fluorescent in situ hybridization of the buccal mucosa found a 22q11 deletion,
confirming the diagnosis. He showed the characteristic facial abnormalities, Hypo-parathyroidism
and congenital heart disease associated with the disorder.
Immunity against which of the following factors is usually not absent in the patients with such a
disorder?
1)
2)
3)
4)
5)

Viral infections
Intracellular bacteria
Extracellular bacteria
Intracellular fungi
Vaccines

The correct answer is choice C.

Children with DiGeorge syndrome, or congenital thymic aplasia, are born without a thymus. The
immune defect includes a profound depression of T-cell numbers and absence of T-cell responses.
Since patients lack the T-cell component of the cell-mediated immune system, they are generally
able to cope with infections of extracellular bacteria, that can be mediated through phagocytosis
and complement mediated lysis but they cannot effectively eliminate intracellular pathogens.

Their lack of functional cell-mediated immunity results in repeated infections with viruses (choice A).
Also, intracellular bacteria, and fungi (choice B and D) can cause serious complications, since any
pathogen for which the normal immune response requires clearance through antigen processing
and presentation is needed can evade the immune response.

The severity of the cell-mediated immunodeficiency in these children is such that even the
attenuated virus present in a vaccine (choice E), capable of only limited growth in normal
individuals, can produce life-threatening infections.

The most common symptoms of children with DiGeorge syndrome include the following:

Congenital heart disease (40% of individuals), particularly conotruncal malformations


(tetralogy of Fallot, interrupted aortic arch, ventricular septal defect, and persistent truncus
arteriosus)

palatal abnormalities (50%), particularly velopharyngeal incompetence (VPI), submucosal


cleft palate, and cleft palate; characteristic facial features (present in the majority of
Caucasian individuals) including hypertelorism.

learning difficulties (90%) but broad range

hypocalcemia (50%)(due to hypoparathyroidism)

significant feeding problems (30%)

renal anomalies (37%)

hearing loss (both conductive and sensorineural) (Hearing loss with craniofacial syndromes)

laryngotracheoesophageal anomalies

growth hormone deficiency

autoimmune disorders

seizures (without hypocalcemia)

skeletal abnormalities

Hypersensitivity refers to undesirable (damaging, discomfort producing and sometimes fatal)


reactions produced by the normal immune system. Hypersensitivity reactions require a presensitized (immune) state of the host. Hypersensitivity reactions can be divided into four types: type
I, type II, type III and type IV, based on the mechanisms involved and time taken for the reaction.
Frequently, a particular clinical condition (disease) may involve more than one type of reaction.
Which of the following is an example of type 4 hypersensitivity reaction:
1)
2)
3)
4)
5)
6)
7)

Coagulase test
Mantoux test
Schick test
Eleks test
A and B
A and B and C
All of the above are correct (A, B, C and D)

The correct answer is choice B

Classically, immune reactions have been organized by the Gel and Coombs classification. Using
this system:

A type 1 reaction or immediate hypersensitivity, is mediated by IgE and histamine

A type 2 reaction is antibody mediated

A type 3 reaction is mediated by immune complexes

A type 4 hypersensitivity reaction or delayed type hypersensitivity is mediated by sensitized


T cells.

The tuberculin test, also known as the Mantoux test, is mediated by delayed type hypersensitivity. A
small amount of immunogenic protein from the mycobacterium is injected under the skin. If the
individual has previously been exposed to TB and has developed immunological memory to the
bacterium, a T cell-mediated response ensues.

Table 1: Delayed hypersensitivity reactions review

Table 2. Comparison of Different Types of hypersensitivity

In the coagulase test, coagulase found on the cell walls of certain types of Staph organisms binds
to fibrinogen, causing it to precipitate or to agglutinate the organisms.

The Schick test is no longer used. It was devised to differentiate people who were sensitive to
diptheria toxin from those who had sufficient immunity to protect them from adverse effects. A
small amount of diptheria toxin would be injected intracutaneously and evaluated after 48 hours.
An unprotected individual would have developed an inflammatory reaction by that point; an

individual with sufficient protective immunity would not have an inflammatory reaction.

Hypersensitivity refers to the excessive, undesirable (damaging, uncomfortable and sometimes


fatal) reactions produced by the normal immune system. Hypersensitivity reactions require a presensitized (immune) state of the host. Hypersensitivity reactions can be divided into four types: type
I, type II, type III and type IV, based on the mechanisms involved and time of onselt for the reaction.
Frequently, a particular clinical condition (disease) may involve more than one type of reaction.
Of the following, which hypersensitivity type is incorrectly matched to the condition?
1) Type I hypersensitivity reaction and anaphylaxis
2) Type III hypersensitivity reaction and acute post-transfusion hemolysis
3) Type III hypersensitivity reaction and serum sickness
4) Type IV hypersensitivity reaction and a Mantoux test of >15 mm induration diameter
5) Mixed type (II and III) hypersensitivity reaction and systemic lupus erythematosis

The correct answer is choice B

An acute hemolytic reaction to a blood transfusion is based on the reactivity of the donor antibodies
to the red blood cell antigens of the host--and therefore a Type II reaction. It is generally an
ABO incompatibility, though other blood group antigens may play a role. Major symptoms include
fever, chills, back pain and heamturia. It is a medical emergency, the most common complication

being acute renal failure.

Type II- Antibody mediated hypersensitivity. Type II hypersensitivity ischaracterized by opsonization


and phagocytosis. Prototypic disorders include auto-immune hemolytic anemias, Goodpasture
Syndrome, Graves' Disease, Myesthenia gravis, Type II diabetes, pemphigus and acute
rheumatoid arthritis. It is Th1 mediated.

The other choices are correctly matched. (Choices A,C,D and E).

Serum sickness occurs with, for example the use of a xenogeneic antivenom for the treatment of a
snake bite (Choice C). A Mantoux test (Choice D) is used to test for reactivity to Mycobacterium
tuberculosis and is considered positive if the indurated zone is >15mm.

A 50-year-old man with end stage renal disease from long-standing diabetes mellitus receives a
cadaveric renal transplant. A month later, he experiences increasing malaise. His urine output
decreases. Laboratory studies show a rising serum urea nitrogen and creatinine. The allograft is
biopsied and seen microscopically to be undergoing destruction by cells which are recognizing graft
cells expressing class I HLA antigens.
Which of the following cells is most likely to mediate acute cellular rejection of solid organ
transplants?
1)
2)
3)
4)
5)

CD4 lymphocytes
NK cells
CD8 lymphocytes
Macrophages
Neutrophils

The correct answer is choice C

Transplant rejection occurs when a transplanted organ or tissue is not accepted by the body of the
transplant recipient. There are three types of transplant rejections that have different time frames
and are mediated by different mechanisms: hyperacute, acute, and chronic rejection.

Hyperacute rejection is a complement-mediated response in recipients with pre-existing antibodies


to the donor (for example, ABO blood type antibodies). Hyperacute rejection occurs within minutes
and the transplant must be immediately removed to prevent a severe systemic inflammatory
response. Rapid agglutination of the blood occurs. This is a particular risk in kidney transplants so a
prospective cytotoxic crossmatch is performed prior to kidney transplantation to ensure that
antibodies to the donor are not present. Hyperacute rejection is analogous to a blood transfusion
reaction as it is a humoral-mediated immune response.

Acute rejection usually begins one week after transplantation (as opposed to hyperacute rejection,
which is immediate). The risk of acute rejection is highest in the first three months after
transplantation. However, acute rejection can also occur months to years after transplantation. A
single episode of acute rejection is not a cause for concern if recognized and treated promptly, and
rarely leads to organ failure. It is caused by mismatched HLA, which are present on all cells of the
body. There are a large number of different alleles of each HLA, so a perfect match between all
HLA in the donor tissue and the recipient's body is extremely rare.

Tissues such as the kidney or the liver which are highly vascularized, are often the earliest victims
of acute rejection. In fact, episodes of acute rejection occur in around 10-30% of all kidney
transplants, and 50 to 60% of liver transplants. Damage to the endothelial lining of blood vessels is
an early predictor of irreversible acute transplant rejection. CD8+ T-cells are involved in the
rejection mechanism. The T-cells cause cells in the transplanted tissue to lyse, or produce

cytokines that cause necrosis of the transplanted tissue.

Chronic rejection occurs many years after transplantion and is due to a poorly understood chronic
inflammatory and immune response against the transplanted tissue. Typically, there is fibrosis of
the internal blood vessels of the transplanted tissue.

CD4 lymphocytes (choice A) are involved in humoral-mediated immune response. They are
involved in chronic rejection of organs.

NK cells (choice B) are a type of cytotoxic lymphocyte that constitute a major component of the
innate immune system. NK cells play a major role in the rejection of tumors and cells infected by
viruses. They kill cells by releasing small cytoplasmic granules of proteins called perforin and
granzyme that cause the target cell to die by apoptosis. They are not involved in acute tissue
rejection mechanisms.

Macrophages (choice D) are phagocytes, acting in both non-specific defense (innate immunity) as
well as to help initiate specific defense mechanisms (adaptive immunity). Their role is to
phagocytose (engulf and then digest) cellular debris and pathogens either as stationary or as
mobile cells, and to stimulate lymphocytes and other immune cells to respond to the pathogen.

They are involved in chronic rejection of organs.

Neutrophils (choice E) are the most abundant type of white blood cells. They are active during the
acute phase of inflammation, particularly as a result of bacterial infection and some cancers.
Neutrophils are one of the first-responders of inflammatory cells to migrate toward the site of
inflammation, firstly through the blood vessels, then through interstitial tissue, following chemical
signals (such as Interleukin-8 (IL-8) and C5a) in a process called chemotaxis. They are the
predominant cells in pus, accounting for its whitish/yellowish appearance.They are not involved in
acute tissue rejection mechanisms.

The second pregnancy for a 23-year-old woman appears uncomplicated until ultrasound performed
at 19 weeks shows hydrops fetalis. The fetal organ development is consistent for 19 weeks, and no
congenital anomalies are noted. Her first pregnancy was uncomplicated and resulted in the birth of
a normal girl at term. The current pregnancy ends with birth of a baby boy at 32 weeks gestation.
On examination the baby has marked icterus. Laboratory studies show a hemoglobin of 7.5 g/dL,
and an exchange transfusion is performed.
Which of the following immunologic mechanisms best explains this infant's findings?
1) IgM deficiency
2) Complement mediated cell destruction
3) Immune complex formation

4) Type 1 hypersensitivity
5) Anti-receptor antibody

The correct answer is choice B.

Until the advent of routine immunization of Rh negative mothers, most immune hydrops was due to
alloimmunization, resulting in the complement-mediated lysis of the infants red blood cells. This is
termed a type II hypersensitivity reaction. Currently, most cases of hydrops are non-immune.
However, immune hydrops continues to occur; exposure to foreign RBC on contaminated needles
in illicit drug use has been shown to be responsible for about 20% of current Rh sensitization.
These women are particularly at risk. One study showed that the incidence of immune hyrops in
their infants was 80%.

While ultrasound can identify fetal hydrops, it cannot determine the fetus hematocrit. Doppler blood
flow analysis is similarly unable to determine hematocrit. At-risk pregnancies are monitored with
repeated analysis of bilirubin levels in the amniotic fluid. Transfusion is the recommended
treatment.

Type II Hypersensitivity Review

It is also known as cytotoxic hypersensitivity and may affect a variety of organs and tissues. The
antigens are normally endogenous, although exogenous chemicals (haptens) which can attach to
cell membranes can also lead to type II hypersensitivity. Drug-induced hemolytic anemia,
granulocytopenia and thrombocytopenia are such examples. The reaction time is minutes to hours.
It is primarily mediated by antibodies of IgM or IgG class and complement. Phagocytes and K cells
may also play a role (ADCC).

The lesion contains antibody, complement and neutrophils. Diagnostic tests include detection of
circulating antibody against tissues involved and the presence of antibody and complement in the
lesion (biopsy) by immunofluorescence. The staining pattern is normally smooth and linear, such as
that seen in Goodpastures nephritis (renal and lung basement membrane) and pemphigus (skin
intercellular protein, desmosome).

Treatment involves anti-inflammatory and immunosuppressive agents.

Table 1. Comparison of Different Types of hypersensitivity

A 41 year old female presented with complaint of excessive sweating, palpitations and unexplained
weight loss of about 5 kg in 2 months. She also complained of increased nervousness and anxiety.
Examination revealed obvious enlargement of thyroid. Her resting pulse rate was high (155/min)
and she was diaphoretic as you examined her.
Laboratory tests revealed raised levels of serum T3 (4nmol/l; normal 1.8-2.9 nmol/l), T4 was also

high at 45nmol/l (normal 9-23 nmol/l). Thyroid secreting hormone was normal at the lower end
(0.4mU/l) (normal range 0.5-5.0 mU/l). Thus, pituitary over activity was ruled out. Circulating
antibodies to thyroid peroxidase were high (1:3000; 200iu/ml). A diagnosis of Graves' disease was
made and the antithyroid drug, carbimazole, was given to control her thyrotoxicosis.
The immunological basis of Graves disease is related to which of the following?
1)
2)
3)
4)
5)

Expression of class I MHC on thyroid acinar cells


Inappropriate expression of class II MHC on thyroid acinar cells
Autoimmunity against acetylcholine receptors
Inability to absorb iodine
None of the above are true

The correct answer is choice B.

The synthesis of two thyroid hormones, thyroxine (T4) and triiodothyronine (T3) is carefully
regulated by the thyroid-stimulating hormone (TSH) produced by the pituitary gland. A patient with
Graves disease produces auto-antibodies that bind the receptor for TSH and mimic the normal
action of TSH, activating adenylate cyclase and resulting in production of the thyroid hormones.
Since the binding of auto antibodies is permanent, the thyroid remains overstimulated. The thyroid
acinar cells from patients with Graves disease express class II MHC molecules on their
membranes. This inappropriate expression of class II MHC molecules, which are normally
expressed only on antigen presenting cells, may serve to sensitize T Helper cells to peptides

derived from the beta cells or thyroid cells (such as the TSH receptor), allowing activation of B cells
or T cytotoxic cells or sensitization of TH1 cells against self-antigens.

Class I MHC genes encode glycoproteins expressed on the surface of nearly all nucleated cells; the
major function of the class I gene products is presentation of peptide antigens to T cytotoxic cells.
Thus, thyroid acinar cells expressing MHC class I (choice A) would not cause Graves disease.

Autoimmunity against acetylcholine receptors (choice C) is a hallmark of Myasthenia gravis in


which antibodies that bind the acetylcholine receptors on the motor end-plates of muscles, blocking
the normal binding of acetylcholine and also inducing complement mediated lysis of the cells. This
results in progressive weakening of the skeletal muscles.

Deficiency of iodine causes hypothyroidism because of decreased thyroid hormone production and
is known as the Goiter. Goiter may also be caused due to autoimmunity against a number of thyroid
proteins, including thyroglobulin, both of which are involved in the uptake of iodine (Choice D)
commonly called Hashimotos thyroiditis.

A 37-year-old woman who was admitted to intensive care unit for respiratory distress associated

with flaccid tetraplegia. Two weeks ago, she had fever, weakness, fatigue, and a cough. A
nasopharyngeal swab was performed, and reverse-transcription polymerase chain reaction (RTPCR) confirmed infection with pandemic flu virus A (H1N1). Ten days later, she suffered muscular
weakness of the lower limb, which then progressed to the upper limb. At admission, she had normal
consciousness, flaccid tetraplegia with complete areflexia, and symmetric paresthesia of the lower
limb. A few hours later, she developed severe dyspnea with impaired coughing, accumulation of
airway secretions, nasal voice, swallowing difficulties, and severe hypoxemia requiring mechanical
ventilation. During the first 2 days, episodes of bradycardia and hypertension were recorded on
hemodynamic monitoring. Electromyography showed reduction in motor conduction velocity and
prolonged distal latencies, which were more severe in the lower limb.
Subsequently, five consecutive plasma exchanges were performed, and the patient improved
progressively. She was free from mechanical ventilation within 2 weeks, and neither swallowing
problems nor dysautonomia abnormalities were reported. The patient was discharged from the
intensive care unit within 45 days.
Which of the following best describes the above scenario?
1)
2)
3)
4)
5)

Amyotrophic lateral sclerosis (ALS)


Guillain-Barr syndrome
Lambert-Eaton myasthenic syndrome (LEMS)
Myasthenia gravis
Inclusion body myositis

The correct answer is choice B.

As described above the classical symptom of Guillain-Barr syndrome include varying degrees of
weakness or tingling sensations in the legs. In many instances, the weakness and abnormal
sensations spread to the arms and upper body. These symptoms can increase in intensity until the
muscles cannot be used at all and the patient is almost totally paralyzed. In these cases, the
disorder is life-threatening and is considered a medical emergency. The patient is often put on a
respirator to assist with breathing.

All forms of GuillainBarr syndrome are due to an immune response to foreign antigens (such as
infectious agents) that are mistargeted at host nerve tissues instead. The targets of such immune
attack are usually gangliosides. The most common antecedent infection is the bacterium
Campylobacter jejuni. In 66% of patients, history of respiratory or gastrointestinal infection within 6
weeks preceding onset of the disease is found. Some cases are triggered by the influenza virus, or
by an immune reaction to the influenza virus. In adults, plasmapheresis is considered the gold
standard for treatment of the most severe cases of Guillain-Barre syndrome.

Amyotrophic lateral sclerosis (ALS) (choice A) is the most common type of adult-onset motor
neuron disease. Adult-onset motor neuron diseases are a group of neurologic disorders that are

characterized primarily by progressive degeneration and loss of motor neurons. ALS involves upper
and lower motor neurons resulting in progressive muscle weakness, atrophy, and fasciculations.
The disease can be divided into familial and sporadic forms. Familial ALS nearly always is
transmitted in an autosomal dominant pattern and a mutation for the gene Cu/Zn superoxide
dismutase on chromosome 21or ALS2, has also been found to be associated with the disease.
Current research into the mechanisms resulting in sporadic types of ALS has focused on
excitotoxicity. This may occur due to overactivation of glutamate receptors, autoimmunity to calcium
ion channels, oxidative stressors linked to free radical formation, or even cytoskeletal abnormalities
such as intracellular accumulation of neurofilaments.

Lambert-Eaton myasthenic syndrome (LEMS) (choice C) is a rare condition in which weakness


results from an abnormality of acetylcholine (ACh) release at the neuromuscular junction. LEMS
results from an autoimmune attack against voltage-gated calcium channels (VGCC) on the
presynaptic motor nerve terminal. Cancer is present when the weakness begins or is later found in
40% of patients with LEMS. Symptoms usually begin insidiously. Many patients have symptoms for
months or years before the diagnosis is made. Weakness is the major symptom, with proximal
muscles more affected than distal muscles (especially in the lower limbs).

Myasthenia gravis (MG) (choice D) is an acquired autoimmune disorder characterized clinically by


weakness of skeletal muscles and fatigability on exertion. The antibodies in myasthenia gravis are
directed toward the acetylcholine receptor (AChR) at the neuromuscular junction (NMJ) of skeletal
muscles. Myasthenia gravis is characterized by fluctuating weakness increased by exertion.
Weakness increases during the day and improves with rest. Presentation and progression may

vary.

Inclusion body myositis (IBM) (choice E) is an inflammatory muscle disease, characterized by


slowly progressive weakness and wasting of both distal and proximal muscles, most apparent in the
muscles of the arms and legs, involvement of the knee extensors, ankle dorsiflexors, and
wrist/finger flexors is characteristic of sporadic IBM, dysphagia is common, as is mild facial
weakness.

You might also like